Sei sulla pagina 1di 69

Earth Geometry

ISAIAH 40:22

As a mathematician, you view the world looking for regularity and order or the lack of order and regularity

BY T.H Musondela

1 by T.H Musondela
INTRODUCTION

When the world celebrated the start of the year 2000, on New Year’s Eve all eyes were on the
small island of Tonga in the south Pacific, because this was the first place to experience the
New Year. Every day the sun rises first on Tonga, then Fiji, New Zealand and eastern
Australia, so this area experiences a new day before the rest of the world. How is this possible

To understand the reason, we must study the geometry and geography of the world and
understand the way distances and times are measured on the Earth’s surface—and take into
account the fact that the Earth is a sphere rather than a flat surface. As long ago as the 2nd
century BC, the Greek astronomer Hipparchus developed a coordinate system for locating
positions on the Earth using angles of latitude and longitude. This system is still used in air
and sea navigation today, as is the imperial unit called the nautical mile for measuring and
charting distances across the globe. We will learn more about Earth measurement in this
chapter, including the use of standard time zones for places around the world. In this topic we
shall take the Earth as a sphere rather than a flat surface of Radius 6370 or 3437 nautical
miles.

OBJECTIVES
By the end of the lesson, the learner should be able to
 define the great and small circles in relation to a sphere.
 identity and define latitudes and longitudes.
 locate a place on the earth’s surface in terms of latitudes and longitudes.
 Sketch and label the latitude and longitude of a given place.
 calculate the distance between two points in nautical miles and kilometers
(i) along a great circle (ii) along a small circle,
 Calculate time in relation to longitudes.
 Calculate speed in knots and in kilometres per hour. Convert nautical miles
to kilometers and vice versa.

2 by T.H Musondela
TEACHING AIDS
The following items are useful teaching and learning aids for understanding the Earth
geometry concepts introduced in this chapter
 World globe, atlas, orange and knife to cut it with, tennis ball, golf ball or
basketball, black markers or rubber bands.
 Wire model should be used to help pupils grasp the concept of the circles of latitude
and longitude.

THE MAIN IDEAS ARE:


 Great circles
 Small circles
 Position on the Earth (Latitude/Longitude)
 Nautical miles
 Shortest distance between two points on a great circle
 Distance between two points on a small circle
 The shape of the Earth closely resembles a flattened sphere with an equatorial radius of
6370 km.
Vocabulary:
Latitude, longitude, Line of latitude, line of longitude (or meridian), co-latitude, equator, zero
meridian ("prime meridian"), eastern and western hemispheres, (local) noon meridian, local
time (LT), time zones, international date line, universal time (UT; of "Greenwich Mean
Time" GMT), nautical mile, knots.

3 by T.H Musondela
GREAT AND SMALL CIRCLES
GREAT CIRCLES
 A great circle is any circle on the surface of a sphere whose plane passes through the
center of the sphere. There are an infinite number of great circles can be drawn on a
sphere.
 Great circles are used in the calculation of distance between two points on a sphere.
Thus, a great circle is a circle with the greatest possible diameter on the surface of a
sphere.
 All circles of longitudes are great circles. The equator is the only circle of latitude
which is a great circle and also the only great circle whose plane is perpendicular to the
polar axis
 The radius of Great circles is approximately 𝟔𝟑𝟕𝟎 kilometers (km) or 3437 nautical
miles (nm).
SMALL CIRCLES
 A small circle is produced by passing a plane through any part of the sphere other than
the center. Thus any circle on the surface of a sphere whose plane does not pass
through the center of the sphere is called a small circle. Parallels of latitude are all
small circles, except for the equator.

4 by T.H Musondela
PARALLELS OF LATITUDE AND LONGITUDES

I have this song written up with a picture of the globe and the lines of latitude, comparing the
lines of latitude to a ladder.
Latitude
I'm climbing the Ladder of Latitudes, latitudes, latitudes
I'm climbing the ladder of latitudes, all around the world.
Longitude
(We stretch our arms from up to down to indicated going from pole to pole
It's a LONG, LONG way from pole to pole, pole to pole, pole to pole
It's a LONG, LONG way from pole to pole, so we call those LONGITUDES.

2.3.1. PARALLELS OF LATITUDE

 Lines of latitude are imaginary lines which run in an east-west direction around the
world. They are also called parallels of latitude because they run parallel to each other.
Latitudes are measured in degrees (°).
Parallels are related to the horizontal x-axes of the Cartesian coordinate system.
 Latitudes Indicate how far north or south of the equator a particular point is situated.
o North latitude: all points north of the equator in the northern hemisphere
o South latitude: all points south of the equator in the southern hemisphere

5 by T.H Musondela
Important parallels of latitude
 The most important line of latitude is the Equator (0°). The North Pole is 90° North
(90°N) and the South Pole is 90° South (90°S). All other lines of latitude are given a
number between 0° and 90°, either North (N) or South (S) of the Equator.
 Latitude measures the position of a point on the earth's surface in terms of the angular
distance between the equator and the poles. Latitude is measured in angular degrees
from 0° at the equator to 90 ° at either of the poles. A point in the northern hemisphere
40 degrees north of the equator is labeled Lat. 40° N. Forty degrees south of the
equator, the label changes to Lat. 40° S.
 Some other important lines of latitude are the Tropic of Cancer (23.5°N), Tropic of
Capricorn (23.5°S), Arctic Circle (66.5°N) and Antarctic Circle (66.5°S).
 Further the equator is the only parallel of latitude being a great circle. Any other
parallel of latitude is a small circle whose plane is parallel to the plane of the equator.
The equator is also the only great circle whose plane is perpendicular to the polar axis
2.3.2. MERIDIANS OF LONGITUDE
 Lines of longitude are imaginary lines which run in a north-south direction, from the
North Pole to the South Pole .They are measured in degrees (°).
 Meridians of longitude are halves of great circles, connecting one pole to the other.
o All run in a true north-south direction
o Spaced farthest apart at the equator and converge to a point at the poles
o An infinite number can be created on a globe
o Meridians are similar to the vertical y-axes of the cartesian coordinate system.
Important longitude
o The most important line of longitude is the Greenwich meridian.
o It is the meridian passing through the Royal Greenwich Observatory in London
(closed in 1998).

o The Greenwich meridian was adopted as the prime meridian at the International
Meridian Conference in October 1884.

6 by T.H Musondela
o Its upper branch (0°) is the reference for measuring longitudes, its lower branch
(180°) is known as the International Dateline. All the lines of longitude are given a
number between 0° and 180°, either East (E) or West.
FEET IN DIFFERENT HEMISPHERE

NB: Located at the eastern edge of London, the British capital, the observatory is now
a public museum and a brass band stretching across its yard marks the "prime
meridian." Tourists often get photographed as they straddle it--one foot in the eastern
hemisphere of the Earth, the other in the western hemisphere

THE INTERNATIONAL DATE


The International Date Line is the 180ºE and west line. This longitude is diametrically
opposite to that of the Greenwich Meridian. The International Date Line is not straight. It
zigzags to avoid land masses. It passes only through water bodies. It marks the place on the
earth’s surface where each new date first begins. The International Date Line curves East of
180º in Bering Strait between Siberia & Alaska. It avoids the Hawaiian, Fiji, and Tonga
Islands.
What is the International Date Line and why is it important?
As one crosses time zones moving westward, at each time zone boundary, one's local time
jumps to one hour earlier. This may move us to an earlier day.
The International Date Line is a special time zone boundary, most of it following the line of
longitude 180°. The line makes sure that in crossing time zones we always match the correct
date at each location. Anyone crossing that line gains a day (crossing westward) or loses one
day (crossing eastward.
Japanese war planes bombed Pear Harbor on Hawaii on December 7, 1941, and in doing so,
pushed the US into war against Japan. Japanese war planes bombed US bases in the
Philippines Islands on December 8. Why did they wait an extra day?. They didn't! Hawaii is
on the other side of the date line from the Philippines. Actually the attacks occurred on the
same day.

7 by T.H Musondela
TERRESTRIAL COORDINATE SYSTEM
The position of an observer on the earth's surface can be specified by the terrestrial
coordinates, latitude and longitude. Any location on a map or globe can be given a pair of
coordinates: The first is the parallel of latitude that it lies on, the second is the meridian of
longitude (𝑳𝒂,𝒍𝒐) .
Positions on the Earth’s surface also can be described by a coordinate system, one involving
latitude and longitude. However, because the Earth is a sphere, we must use a special grid of
lines that run across and down a sphere. The World maps or globes are drawn with both
parallels of latitude and meridians of longitude shown.

GRID OF LINES THAT RUN ACROSS AND DOWN A SPHERE

A place on the surface of the earth is represented by a point. The location of a place A at
latitude x◦ N and Longitude y◦ E is written as 𝑨(𝒙° 𝑵, 𝒚° 𝑬). Thus D (40°N, 60°W),
I(30°S, 20°E) and H (20°S, 50°E). For example, the coordinates of Sydney are 30°S, 150°E.
or Sydney (30°S, 150°E).
The co-ordinates of every point on the earth are always expressed as (Latitude N or S,
Longitude E or W) . Latitude always goes first.

8 by T.H Musondela
DEGREES, MINUTES, AND SECONDS

 These are Angular measurement used in addition to simple plane geometry to specify
location on the earth's surface.
 A circle has 360 degrees, 60 minutes per degree, and 60 seconds per minute. There
are 3,600 seconds per degree. This is based on a sexagesimal scale:
 If a pilot or a ship's captain wants to specify position on a map, (latitude, longitudes)
are the "coordinates" they would use. Actually, these are two angles, measured in
degrees, "minutes of arc" and "seconds of arc." These are denoted by the symbols ( °, ',
" ) e.g. 35° 43' 9" means an angle of 35 degrees, 43 minutes and 9 seconds. A degree
contains 60 minutes of arc and a minute contains 60 seconds of arc--and you may omit
the words "of arc" where the context makes it absolutely clear that these are not units
of time.

 Example: 45° 33' 22" (45 degrees, 33 minutes, 22 seconds).


 It is often necessary to convert this conventional angular measurement into decimal
degrees.
o To convert 45° 33' 22", first multiply 33 minutes by 60, which equals 1,980
seconds.
o Next add 22 seconds to 1,980: 2,002 total seconds.
o Now compute the ratio: 2,002/3,600 = 0.55.
o Adding this to 45 degrees, the answer is 45.55°.
o The earth rotates on its axis once every 24 hours, therefore:
o Any point moves through 360° a day, or 15° per hour.
Use the key °   on your calculator to convert these times to hours and minutes

°

9 by T.H Musondela
Pupil’s activity
1. Use the key ° ° on your calculator to convert these times to hours and
minutes.
(a) 22.7 hours (b) 6.3 hours (c) 4.9 hours (d) 18.4 hours (e) 10.1 hours (f) 15.6 hours
FLAT WORLD MAP (2D)
The Earth is round but it is also very big — it has a circumference of about 40 000 km. This
means that for distances on the Earth's surface of less than 20 km we can treat this Earth as
being 'flat* and use flat-Earth formulas such as Pythagoras*s Theorem to find the distance
between points. Points that lie on the same meridian are on a great circle. We can calculate
the distance between them using the difference in their latitudes.

A B

The diagram below illustrate this grid on a flat world map (2D)
The earth's spherical shape is more difficult to describe than a plane surface. Concepts from
Cartesian coordinate geometry have been incorporated into the earth's coordinate system.
Thus the position of A is (20° S,140°W) (That is 20 degrees south of the equator and 140
degrees west of Greenwich) while B is (20° 𝑆,60°𝐸) (That is 20 degrees south of the equator
and 60 degrees East of Greenwich)

10 by T.H Musondela
ANGULAR DISTANCES: Angular distances are the amount of differences either in
latitudes or in longitude
Difference in longitudes can be found either by adding or subtracting the two longitudes
depending whether both are on the same hemisphere (i.e. either both are either on western
hemisphere or both are on the eastern hemisphere) or are on different hemispheres (i.e. one is
on western hemisphere and the other is on the eastern hemisphere.
RULES TO FOLLOW
 If the two longitudes are on the same hemisphere , the difference in longitude is found
by subtraction the smaller longitude from the larger longitude
 If the two longitudes are on different hemisphere i.e. one is on the western hemisphere
or and the other is on the Eastern hemisphere the difference in longitude is found by
adding the two longitudes in any order
The difference in longitudes is the angle needed for calculating distances along any line
of latitude. It is also used for time calculations.
Example
(a) The coordinates of Sydney and Tokyo, Japan are (34°S, 151°E) and (35°N, 139°E)
respectively. What is the difference in longitude between Sydney and Tokyo?

Solution
Longitude of Sydney is 151°E and Tokyo is on longitude 139°E. Both Sydney and Tokyo
are on the eastern Hemispheres so we subtract.
∴ Difference in longitude = 151° - 139° = 12° Horizontal angular distance

(b) A (80°N, 100°W) and B (70°N, 80°E) are two places on the surface of the earth. Find the
difference in longitudes

Solution
Longitude of A is 100°W and B is on longitude 80 °E. A is on western Hemisphere and B
is on the eastern Hemispheres so we subtract.
∴ Difference in longitude = 100° +80° = 180 °
Mostly when the difference in longitudes is 180, the distance between the two places can
be calculated via either the North Pole or south poles
11 by T.H Musondela
(b) A (80°N, 120 °W) and B (70°N, 85°E) are two places on the surface of the earth. Find the
difference in longitudes

Solution
Longitude of A is 120°W and B is on longitude 85 °E. A is on western Hemisphere and B
is on the eastern Hemispheres so we subtract.
∴ Difference in longitude = 120° +80° = 205 °
Since the difference in longitudes is greater that 180 then 𝟑𝟔𝟎° ‒ 𝟐𝟎𝟓° = 155° will give us
the shortest distance between the two places using the route that closes the International
Date Line.

DIFFERENCE IN LATITUDES
The angular distance between two points on the same great circle (meridian of longitude) can
be found by:
• subtracting the latitudes if the points are on the same side of the equator
• adding the latitudes if the points are on opposite sides of the equator.
The difference in latitude is the angle needed for calculating distances along any line
longitude.
(b) Two points, A and B, on the Earth’s surface are at (30°N, 25°W) and (20°S, 25°W).
Calculate the angular distance between A and B.
Solution

Latitude of A is 34°S and B is on latitude 35°N

∴ Difference in latitude = 30° + 20° = 50° Vertical angular distance


(a) The coordinates of Sydney and Tokyo, Japan are (34°S, 151°E) and (35°N, 139°E)
respectively. What is the difference in latitude between Sydney and Tokyo?

Solution

Lat of Sydney is 34°S and Tokyo is on lati 35°N ∴ Difference in lat = 35° + 34° = 69°

12 by T.H Musondela
PUPIL’S ACTIVITY
1. Calculate the difference in longitude between these cities, and state whether the second city
listed is east or west of the first.
(a) Budapest (47°N, 19°E) and Miami (25°N, 80°W)
(b) Athens (38°N, 23°E) and Paris (49°N, 2°E)
(c) Havana (23°N, 82°W) and Mexico City (19°N, 99°W)
(f) Finley (35°S, 145°E) and Bourke (30°S, 146°E)
2. The city of Durban is at approximately (30°S, 30°E) while Cairo is at approximately
(30°N, 30°E). What is the angular distance between Durban and Cairo?
3. Calculate the difference in latitude between these cities, and state whether the second city
listed is north or south of the first.
(a) Shanghai, (31°N, 121°E) and New York, USA (40°N, 64°W)
(b) Nairobi (1°S, 37°E) and Bangkok, (13°N, 100°E)
(c) Moscow, Russia (55°N, 37°E) and London, UK (51°N, 0°)
(d) Auckland, NZ (37°S, 174°E) and Canberra (35°S, 149°E)
(e) Melbourne (37°S, 145°E) and Cairo, Egypt (30°N, 31°E)
(f) Newcastle (33°S, 151°E) and West Wyalong (34°S, 147°E)
(g) A (20°S, 130°E) and B are (15°N, 130°E).

4 . In each of the following calculate the angular distance between the pairs of points given.

(a) (70°N, 150°E) and (30°N, 150°E) (b) (25°N, 40°W) and (15°S, 40°W)
(c) (64°N, 0°) and (7°S, 0°) (d) (42°S, 97°W) and (21°S, 97°W)
(e) (0°, 60°E) and (0°, 20°W) (f) (10°S, 60°E) and (70°S, 60°E)

5. The city of Durban is at approximately (30°S, 30°E) while Cairo is at approximately


(30°N, 30°E). Find the difference in latitude between Durban and Cairo°
6. P (45°N, 10°W) and Q (15°N, 10°W) are two points on the Earth’s surface.
(a) Calculate the distance between P and Q in nautical miles (M).
(b) Use 1 M =1.852 km to calculate the distance, PQ, correct to the nearest km.

13 by T.H Musondela
ABOUT TIME--LOCAL AND UNIVERSAL
Two important concepts, related to latitude and (especially) longitude are Local time (LT)
and Universal time (UT)
Local time is actually a measure of the position of the Sun relative to a locality. At 12 noon
local time the Sun passes to the south and is furthest from the horizon (northern hemisphere).
Somewhere around 6 am it rises, and around 6 pm it sets. Local time is what you and I use to
regulate our lives locally, our work times, meals and sleep-times.
But suppose we wanted to time an astronomical event--e.g. the time when the 1987 supernova
was first detected. For that we need a single agreed-on clock, marking time world-wide, not
tied to our locality. That is universal time (UT), which can be defined (with some slight
imprecision, no concern here) as the local time in Greenwich, England, at the zero meridian.
LOCAL TIME (LT) AND TIME ZONES
 Longitudes are measured from zero to 180° east and 180° west (or -180°), and both
180-degree longitudes share the same line, in the middle of the Pacific Ocean.
 As the Earth rotates around its axis, at any moment one line of longitude--"the noon
meridian"-- faces the Sun, and at that moment, it will be noon everywhere on it. After
24 hours the Earth has undergone a full rotation with respect to the Sun, and the same
meridian again faces noon. Thus each hour the Earth rotates by 360/24 = 15 degrees.
 When at your location the time is 12 noon, 15° to the east the time is 1 p.m., for that is
the meridian which faced the Sun an hour ago. On the other hand, 15° to the west the
time is 11 a.m., for in an hour's time, that meridian will face the Sun and experience
noon.

TIME ZONES
 As the Earth rotates, different parts of the globe are experiencing day and night. This
means that each meridian of longitude on the Earth’s surface should have a different
time of day. To simplify this, the Earth is divided into time zones.
 Time zones are all calculated in relation to Greenwich. The time on the Greenwich
Meridian is known as Greenwich Mean Time (GMT). Time zones are then stated in

14 by T.H Musondela
terms of the number of hours they are ahead or behind GMT. All places with
longitudes east of Greenwich are ahead of GMT, while all place with longitudes west
of Greenwich are behind GMT. For example, Eastern Standard Time is GMT +10,
meaning that Sydney is 10 hours ahead of GMT. When GMT is noon, EST is 10.00
pm.
 The International Date Line is 12 hours ahead of Greenwich when travelling east and
12 hours behind when travelling west, so this totals 24 hours, or one day. Therefore,
the day changes on either side of the International Date Line.
 All points along the some longitude have the same time.
TIME CALCULATIONS
The time difference between two places is calculated by subtracting the comparative time
with GMT. Once we have calculated the time difference, we are able to calculate the time in
one place given the time in another. To calculate the time in a city further ahead of GMT we
add time, or to calculate the time in a city further behind GMT we subtract time.

It is important to note that, for convenience, places that have almost the same longitude have
the same time. An example of this is Australia’s time zones where all of Queensland, New
South Wales, Victoria and Tasmania are in the same standard time zone although there is a
difference of 12° in longitude from the easternmost and westernmost points in this zone.
These calculations can then be used to calculate the arrival and departure times for
international travel.

WORKING WITH TIME


We work with and calculate time in many ways in our everyday life. The ability to accurately
add and subtract units of time is an essential skill. Some shipboard operations where this skill
is employed include calculating the times of arrivals, optimum tides and currents, and
celestial events such as the time of sunrise or sunset. In addition, having a solid grasp of
calculating time is crucial to entering many nautical publications. In the maritime industry
we always use the 24-hour clock, or military time. In military time, a colon is omitted in the
military twenty-four-hour system of expressing time:

15 by T.H Musondela
FORMAT OF WRITING TIME
Note:
- Figures are used in designation of time with a.m. or p.m.: at 2:30 p.m. 12:00 a.m. (noon)
12:00 p.m. (midnight) at 9:15 in the morning
- Never use o’clock with either a.m. or p.m. or figures, and spell out in words eight o’clock
- A colon is used between hours and minutes: 4:30 p.m. 16:00 hrs.
- Decimal (dot) can also be used for the time of day (but not as popular as colon) 3.15 p.m.
15.15 hours
- Do not use decimals for a period of time: e.g. 3.15 hours, but spell it out as: three hours and
fifteen minutes, or 3 hours and 15 minutes.
- A colon is omitted in the military twenty-four-hour system of expressing time:
WAYS OF TELLING THE TIME
There are two ways of telling the time:
 The 12-hour clock runs from 1am to 12 noon and then from 1pm to 12 midnight. a.m.
is from ante meridiem, which means before midday.
 The 24-hour clock uses the numbers 00:00 to 23:59 (midnight is 24:00 or 00:00). p.m.
is from post meridiem, which means after midday.
24-HOUR CLOCK
The 24-hour clock system shows time for a 24-hour period of time. On the 24-hour clock, the
first 12 hours of the day are numbered 1 to 12. The next hour is 13. The next hour is 14, and
so on. The military uses the 24-hour clock system to avoid confusion in determining if a
given time is a.m. or p.m. For this reason, the 24-hour clock is often referred to as military
time. A colon is omitted in the military twenty-four-hour system of expressing time:
WHAT IS MILITARY TIME?
Military time is a popular expression of the 24-hour clock convention. This convention
divides one day in 24 hours and describes that one day starts at 00 00 hours and progresses
with each hour until it is ending at 2400 hours or starts at 00 00 hours again. This system of
telling time is used in most countries and there’s even an international standard for this

16 by T.H Musondela
convention from the ISO (International Organization for Standardization) under ISO 8601.In
the United States, Canada and some other counties many people refer to the 24-hour clock as
military time.

Process: SAP is a global software product that is utilized in many countries that
operate on military time or a 24-hour clock. See the chart below to enter the
correct start and end times for each travel document.
TIP: When entering the military time into the travel document, the colon does not
need to be entered. For example, if the trip begins at 1PM, enter 1300 into the start
time field and the system will automatically change it to13:00. to Military

MILITARY TIME CONVERSION CHART


Civilian Time (12-hour clock) to Military Time (24-hour clock)
Conversion

REGULAR TIME MILITARYTIME REGULAR TIME MILITARYTIME


2:00 AM 02 00 NOON 12 00
3:00 AM 03 00 1:00 PM 13 00
4:00 AM 04 00 2:00 PM 14 00
5:00 AM 05 00 3:00 PM 15 00
6:00 AM 06 00 4:00 PM 16 00
7:00 AM 07 00 5:00 PM 17 00
8:00 AM 08 00 6:00 PM 18 00
9:00 AM 09 00 7:00 PM 19 00
10:00 AM 10 00 8:00 PM 20 00
11:00 AM 11 00 8:55 PM 20 55
10:53 a.m 10 53 Midnight 2400 or 0000
8. 9:12 p.m. 21 12 5:25 a.m. 05 25
. 4:10 p.m. 1610 2:22 p.m. 14 22
11:29 p.m. 23 29 11:29 a.m 11 29

EXAM TIP: Pupils should take note of the format used in the examiners question.
If the examiner uses military time, only the answers without a colon will be
accepted.

17 by T.H Musondela
ADDING AND SUBTRACTING TIME
Adding Time: (You may go over 60 minutes, therefore remember to add one hour)
In earth geometry addition of time is a skill that must be known. For places to the east of the
given time we add time.
Example 1: Find the time 12 33 + 06 21
Solution The total number of minutes is
not over 60 minutes; therefore
1 2 3 3
remember to add one hour
+ 0 6 2 1
1 8 5 4

Example 2: Find the time 17 42 + 04 27 The total number of minutes goes


over 60 minutes; therefore
Solution remember to add one hour to 21
to make 22 and the number of
1 7 4 2
minutes remain 09
+ 0 4 2 7
21 6 9 2169 = 22 09 hours

Example 3: Find the time 06 56 + 1950 106


60 = 1 Remainder 46 the 1 is
Solution added to 25 to give us 26. So now
we have 26 46. So now 26 46 -24
0 6 5 6 (May 1st)
00 = 02 46
+ 1 9 5 0
2 5 106 26 46 = 0246 (May 2nd next day)

SUBTRACTING TIME:
Subtracting Time: (You may need to borrow one-hour or 60 minutes)
For places to the west of the given time we subtract time to calculate the time zone
Example 1: Find the time 1233 - 0621
Solution This is straight forward since
33 is greater than 21 and 12 is
1 2 3 3 greater that 06.
- 0 6 2 1
0 6 1 2

18 by T.H Musondela
Example 2: Find the time 1737 - 09 54
Solution
Borrow 60 minutes from 17 00 and add to the 37 min =97 min. You can then subtract 54
minutes from 97. 97 – 54 = 43)
We borrow 1 hour which is 60
1 7 3 7 16 97 minutes and add it to 37 to give a
+ 0 9 5 4 ⟹ - 09 54 sum of 97. For the sake of
calculation now we have 16 97
07 43

Example 3: Find the time 02 39 (May 1st) - 19 51


Solution
Solution: In this problem we cannot subtract 1900 from 0200. 0200 is in the morning of the
next day, so we need to add 24 hours to the 0200 = 2600. Now we can subtract 19 from 26.
0648 (May 1st)
0 2 3 9 26 39 (May 2nd) 26 99
+ 1 9 5 1 ⟹ - 09 51 ⟹ - 09 51
07 48
EXERCISE
1. Add the following time problems:

(a). 03 44 May 15th (b). 16 04 May 15th (c). 11 08 May 15th


+ 01 33 +14 14 +08 27

(c). 09 35 May 15th (d). 01 50 May 15th (d). 11 50 May 15th


+07 55 +21 15 +06 45

2. Solve the following time problems:

(a). 11 08 May 15th (b). 16 04 May 15th (c). 11 55 May 15th


- 08 37 -14 14 -08 27

19 by T.H Musondela
(c). 13 19 May 15th (d). 02 16 May 15th (d). 09 12 May 15th
- 04 38 +23 23 - 16 35

3. A plane left an airport at 13 23. It arrived at its destination in the same time zone at 1610.
How long was the flight?

4. Mr. Larson put a roast in the oven at 14 45. He cooked the roast for 3 hours 48 minutes.
What time did Mr. Larson take the roast out of the oven?

TO FIND THE TIME AT A GIVEN PLACE

To find the time at a given place

 Calculate the difference in longitudes.


 Find the time difference by Converting the difference in longitudes into hours and
minutes using 1° = 4minutes or 15° = 1 hour
𝑫𝒊𝒇𝒇𝒆𝒓𝒆𝒏𝒄𝒆 𝒊𝒏 𝒍𝒐𝒏𝒈𝒊𝒕𝒖𝒅𝒆𝒔
Difference in time = 𝟏𝟓
hrs

Add or subtract the time difference to/ from the time in the given time.

For places to the east we add time, for places to the west we subtract time to calculate the
time zone. Care must be taken when crossing the International Date Line as you will
need to add a day when travelling east or subtract a day when heading west.

Time differences need to be used when calculating departure time and arrival time for
international travel. When daylight saving time applies, we add one hour to the standard time.

Example
1. Calculate the time in Los Angeles (34°N, 120°W) when it is 08 00 am on Wednesday in
Sydney (33°S, 150°E).

Solution
Longitude difference =150° + 120° = 270°,
𝟐𝟕𝟎
Time difference = 𝟏𝟓
=18 hours
Time in Los Angeles 08 00 +18 00 = 26 00
Time in Los Angeles 26 00 - 2400 = 02 00 Tuesday

20 by T.H Musondela
2. A plane leaves London (50°N, 0°) at 09 00 Sunday, London time, and flies to Sydney
(33°S, 150°E). The flight takes 20 hours. Calculate the time in Sydney when the plane arrives.

Solution
150
Longitude difference = 150°- 0° = 150°, Time difference = 15
=10 hours

Calculate the time in Sydney when the plane is departing London by adding the time
difference. When the plane leaves London at 09 00,
Time in Sydney =0 9 00 + 10 00 hours = 19 00 Sunday
Add the flying time to calculate the time when the plane lands. Plane arrives at 19 00 +
20 00 = 39 00 ∴ 3900 ‒ 2400 = 15 00
Time in Sydney 15 00 Monday

EXERCISE

1. A plane leaves Z (47°N, 18°W) at 07 20 hours and flies for 3 hours to R (47°N, 19°E).
Calculate the time the plane arrived at R.

2. Lusaka is on longitude 30°E. What is the local time in Lusaka if it is 11 00hours G.M.T?

3. Mogadishu and Kinshasa are on longitudes 𝟒𝟓°𝐄 and 𝟏𝟓°𝐄. Find the time in Kinshasa
when it is 11 30 hours in Mogadishu.

4. A soccer match kicked of at 14 30 hours at A (𝟑𝟑°𝐍,𝟑𝟎° 𝐖). What would be the time at
town B (𝟏𝟐° 𝐒,𝟑𝟐°𝐄)

5. Town A is on (0°, 29° E) and town B is on (0°, 59°W), find the time at B when it is 1230
at A

6. Point A is (40°S, 45°W) and Point B is (30°S, 48°E). An Aircraft took off from Point B at
07 35 hours going to A, the journey took 3hours. Calculate the time it arrived at Point B.

7. Point C is (60°S, 55°W) and Point D is (60°S, 50°E). An Aircraft took off from Point B at
07 35 hours going to D, the journey took 3hours 15 minutes. Calculate the time it arrived at
Point C.

8. The coordinates of two points on the Earth’s surface are given by the coordinates A (50°N,
120°W) and B (30°S, 120°W). Calculate the time taken for a ship to sail the shortest distance
between these two points at an average speed of 40 knots.

21 by T.H Musondela
9. The difference in longitude between town A and B is 105°. B is west of A. Given that a
football match kicks off at 08 37 at A, find the local time at B when the match begins.
10. Point A is (30°S, 77°W) and Point B is (30°S, 32°W). An Aircraft took off from Point A
at 13 00 hours, the journey took 7hours.

(a) Calculate time at Point B at the time the Aircraft took off from Point A

(b) Calculate the time it arrived at Point B.

11. A plane leaves Z (0°, 30 °W) at 06 00 hours and flies due eat for 3 hours to R (0°, 50°E).

(a) Calculate its average speed.

(b) Calculate the time the plane arrived at R.

12. Point A is (65°S, 45°W) and Point B is (65°S, 30°E). Find the time at B when it the local
tine at A is 09 00.

DISTANCES ON THE EARTH

The shortest distance between two points is a straight line. However, we cannot tunnel
through the Earth to make a straight-line path between positions on the surface of the Earth.
The shortest distance on the surface of the Earth will be directly above the line going
through the Earth. It will be a curve following a great circle route, because the straight line
is a chord of the great circle directly above it. We are going to model the earth as a perfect
sphere of radius 𝑅 = 6370 𝑘𝑚 or 3437 nautical miles
DISTANCE IN KILOMETERS ON THE SURFACE OF THE EARTH
In Earth geometry we take it that the earth is a perfect sphere. We ignore mountains and
valleys. Radius of the earth to the 3 significant figures is 6370 Km.

Great circles have a radius of about 6370 km, so their circumference is


C= 𝟐 × 𝝅 × 𝟔𝟑𝟕𝟎 = 𝟒𝟎𝟎𝟐𝟗.𝟏𝟗 𝒌𝒎
1
Travel through an angle of 1 degree on a great circle will involve a distance of 360
the

of the circumference. Dividing 𝟒𝟎𝟎𝟐𝟗.𝟏𝟗 km by 360, we find that:


On a great circle l ° = l l l.19 km

The distance between two consecutive parallels of latitude is equal to about l l l.1918 km, but
the distance between two consecutive meridians of longitude is equal to 111.1918km only at
the equator. Because they are parallel or equidistant throughout 𝟒𝟎𝟎𝟐𝟗.𝟏𝟗 /360 while the

22 by T.H Musondela
distance between two consecutive meridians is 111km only at the equator because meridians
are broadest at the equator & they converge at the poles, and hence they decrease. This means
that

(i) The distance, in kilometers, along any of the great circles (all the meridians and the
equator) where R= 6370 km and 𝜃 is the difference in latitudes is calculated by
𝛉
Distance = 𝟑𝟔𝟎 × 𝟐𝛑𝐑 = 𝟏𝟏𝟏.𝟏𝟗𝟏𝟗 × 𝛉

(i) The distance, in kilometers, along any of the small circles (all the parallel of latitudes
including the equator since𝑐𝑜𝑠 0° = 1) where R= 𝟏𝟏𝟏.𝟏𝟗𝟏𝟗 km, 𝜃 is the difference in
longitudes and lat is the common circle of latitude is calculated by
𝛉
Distance = 𝟑𝟔𝟎 × 𝟐𝛑𝐑𝐜𝐨𝐬 𝐥𝐚𝐭 = 𝟏𝟏𝟏.𝟏𝟗𝟏𝟗 × 𝛉 𝒄𝒐𝒔 𝒍𝒂𝒕

DISTANCES ON GREAT CIRCLES


Example 1: Find the shortest distance between the two towns A (25o S, 30o E) and B (35 o N,
30o E) along the line of longitude

Solution
The towns have a common longitude therefore the distance is on that longitude which is a
great circle with radius R= 6370 km. The angle θ in the formula above is given by the sum or
difference of the latitude. Since we have different direction 25o S and 35o N, we add them,
that is, θ = 25+35 = 60. If it were to be South-South or North-North, we would subtract the
angles (for 25o S and 35o S, θ = 35o–25o=10).
θ
Length of arc: AB = 360 × 2 π R, where R= 6370 km for distances on great circles
60
= 360 × 2 × 3.142 × 6370

= 6671.5133333
= 6700 km (3sf)

23 by T.H Musondela
Example 2: Find the distance between Auckland Island (51°S, 166°E) and Nemea (22°S,
166°E).

Solution
Since the two positions have the same longitude they are on the same meridian, which is a
great circle. The angle travelled is given by the difference in latitudes.
Latitude difference = (51 - 22) ° = 29°
𝟐𝟗
Distance = 𝟑𝟔𝟎 × 𝟐𝛑𝐑 = 𝟏𝟏𝟏.𝟏𝟗𝟏𝟗 × 𝛉
𝟏𝟎
Distance = 𝟑𝟔𝟎 × 𝟐(𝟑.𝟏𝟒𝟐)(𝟔𝟑𝟕𝟎) = 𝟏𝟏𝟏.𝟏𝟗𝟏𝟗 × 𝟐𝟗
= 3224.564777778 = 3224.56477777
= 3220 Km 3 sf

DISTANCES ON SMALL CIRCLES


The analysis is the same as that of the distance on great circles. The only difference is that the
radius, r, of a small circle is not equal to that of the earth (R=6370km) The radius of the small
circle, r = 𝑅𝒄𝒐𝒔 (𝒍𝒂𝒕), where lat is the common latitude. The distance along the
LATITUDES depend of the factor 𝒄𝒐𝒔 𝒍𝒂𝒕

RADIUS OF THE SMALL CIRCLE


As you travel around a line of latitude (small circle) the distance travelled is shorter than the
distance covered on the Equator (great circle) for a given angle. So the small circle is smaller
than the great circle by a factor of 𝒄𝒐𝒔 (𝑳𝒂𝒕𝒊𝒕𝒖𝒅𝒆 𝒂𝒏𝒈𝒍𝒆 ) .

N
B r M
B M
𝑥° 𝑥°

𝑥° 𝑂
C R

24 by T.H Musondela
If we let r be the radius of the circle of latitude and R is the radius of the Earth then MB
is parallel to OC as the result angle COB is equal to angle OBM. And triangle OBM is
the right angled triangle. To find the size of angle OBM we use trigonometry
𝒓
𝑪𝒐𝒔𝒙° = 𝑹 (x is the latitude)

Then r = 𝐑 𝐜𝐨𝐬 (𝐥𝐚𝐭 x°)

Radius of the small circle (r) =Radius of the Equator (R) × cos Lat, where R is the
radius of the Earth which is approximately 6370 km or 3437 nautical miles.

Example 1: Given that latitude is 30°N. Find the radius of its circle, given that R = 6370km

.
Solution
r = 𝑅 𝑐𝑜𝑠 (𝑙𝑎𝑡𝑖𝑡𝑢𝑑𝑒)
= 6370 𝑐𝑜𝑠 (30°)
= 5516.5818
= 5520 km (3sf)

Example 2: A circle of latitude has radius 5200km. Find the latitude given that its situated in
the southern hemisphere, taking R = 6370km.

Solution
𝒓
𝑪𝒐𝒔 𝑳𝒂𝒕° = 𝑹 (x is the latitude)
𝟓𝟐𝟎𝟎
𝑪𝒐𝒔𝑳𝒂𝒕 ° = 𝟔𝟑𝟕𝟎 ,
‒ 𝟏 𝟓𝟐𝟎𝟎
𝑳𝒂𝒕° = 𝐜𝐨𝐬 ( ) = 35.281
𝟔𝟑𝟕𝟎

Therefore, the latitude is 35.3° S.

25 by T.H Musondela
Example 3: Find the length of the circle of latitude 70° , taking R = 6370km as the radius of
the earth

Solution
𝑪 = 𝟐𝝅𝑪𝒐𝒔 𝑳𝒂𝒕°
𝑪 = 𝟐 × 𝟑.𝟏𝟐𝟔 × 𝒄𝒐𝒔 𝟕𝟎
= 13690.751678794664

= 13700 3 sf

Example 4 What is the distance from Woomera (31°S, 137°E) to Kalgoorlie (31°S, 121°E)
along the parallel of latitude?

Solution
Since the two positions have the same latitude they are on the same latitude, which is a
small circle. The angle travelled is given by the difference in longitude. longitude
difference = (137 - 221) ° = 16°
16
Distance = 360 × 2πR = 𝟏𝟏𝟏.𝟏𝟗𝟏𝟗 × 𝛉 =
16 𝟏𝟏𝟏.𝟏𝟗𝟏𝟗 × 𝟏𝟔 × 𝐜𝐨𝐬(𝟑𝟏)°
Distance = 360 × 2(3.142)(6370)cos(31)°
= 1524.9609725270273
= 1524.4754729679
= 1520 Km 3 sf

26 by T.H Musondela
Example 5 : Find the shortest distance along the line of latitude between the towns A (30 o N,
50 o E) and B (30 o N, 40 o W) in Km

Solution
since the towns are on a common latitude 30 o N (which is a small circle) the distance is
along it. Therefore its radius is r, where r = Rcos lat and lat= 30 o (i.e. the common latitude)
and R = 6370km (the radius of the earth). Therefore r = 6370 × cos 30 o.

Also the angle θ is given by the longitude, θ = 50 o + 40 o = 90 o


𝜃
∴ BA = 360 × 2𝜋𝑟 where r = r = 6370 × cos 30 o
90
= 360 × 2 × 3.142 × 6370 × cos30o

= 8666.5500425
= 8670 km

DISTANCE ALONG LATITUDE: Parallels of latitude decrease in length with increasing


latitude.
(a) Circumference of any circle of latitude
Length of parallel at latitude x = (length of equator) * (cosine of x)
EXERCISE
1. Find the radius of its circle of longitude of each of the following latitude in kilometers
Taking R to 6370km.
( a ) Latitude 40° N ( b ) Latitude 56 ° S (c ) Latitude 78° N ( d )Latitude 63° S
2. Find the latitude of each of the following radius
(a) 4583 km in northern hemisphere (b) 3754 km in southern hemisphere
3. Find the circumference of the circle of latitude given below. Take R and π to be 6370km
and 3.142 respectively.
(a) Latitude 45°N (b) Latitude 36° S (c) Latitude 70° S
4. Find the distance in km along a line of latitude between the points A (55°N, 20°W) and B
(55°N, 40°E)
6. Find the distance in nm along the line of latitude from X (40°N, 100°W) and Y (40°N,
65°W)

27 by T.H Musondela
Example 5 : The diagram below is a sketch of the earth and on it are the points P(20°N,80°
E), Q(40°S, 80°E) and R(40° S, 30° E). N
[Use 𝜋 = 3.142 𝑎𝑛𝑑 𝑅 = 6370𝑘𝑚]
(a) Calculate the distance QR in kilometres. 20° N
P
(b) An aeroplane starts from P and flies 40° E 80° E
due west on the same latitude covering a distance
R Q 40°S
of 1 232km to point T.
(i) Calculate the difference in angles between P and T,
S
(ii) Find the position of T.

(a) Difference in longitude 𝜃 = 80° - 30° = 50° S since QR is on latitude


𝜃
𝑄𝑅 = 360° × 2𝜋𝑅𝑐𝑜𝑠 𝑙𝑎𝑡

50°
= 360° × 2 × 3.142 × 6370𝑐𝑜𝑠40°

= 4258.8964 km

≈ 𝟒𝟐𝟔𝟎 𝒌𝒎 (3 𝑠𝑓)
𝜃
(b) (i) 360° × 2 × 3.142 × 6370 cos20° = 1232

111.1919 𝜃cos 20 = 1232


1232
𝜃 = 111.1919cos 20

∴ θ =11.79103106
≈11.8° (1 d p)

(i) Difference in angles between P and T ≈ 𝟏𝟏.𝟖° and both P an Q are on the same
latitude 20°.
Since the longitude of P is on the eastern hemisphere and the direction of the jet is
westward, the longitude of Q =|80 - 𝟏𝟏.𝟖° | = 68.2 ° 𝐸 ∴ 𝑸(𝟐𝟎°𝑺, 𝟔𝟖.𝟐°𝑬)

28 by T.H Musondela
NAUTICAL MILE
A nautical mile (NM) is defined as the distance travelled on a great circle when
moving through 1 minute of arc.
The nautical mile (n mile) is equal to 1 852 km and is used in navigation to measure
the distance travelled on the surface of the Earth. It is defined as the length of an arc of
a great circle subtended by 1” (1 minute: 60 minutes = 1°)

From the definition, we note that there are 360o in a circle and 60 minutes in a degree. So
circumference of any great circle in nautical miles is 360 x 60 = 21600nm. Since
𝟐𝝅𝑹 = 𝟐𝟏𝟔𝟎𝟎 𝒏𝒎
21600 21600
Then R = 2𝜋
= 2(3.142) = 3437.3010 ≈ 𝟑𝟒𝟑𝟕 to the nearest whole number.

Radius of the great circle in nautical mile

R= 𝟑𝟒𝟑𝟕nm

DISTANCE IN NAUTICAL MILES


For sea and air travel, the nautical mile (abbreviated nM) is used instead of kilometres. It has
been used for centuries by sailors and navigators because it is directly related to the latitude–
longitude system

(i) The distance, in nautical miles, along any of the great circles (all the meridians and
the equator) where R= 3437 Nm and 𝜃 is the difference in latitudes is calculated by
𝛉
Distance = 𝟑𝟔𝟎 × 𝟐𝛑𝐑
= 𝟔𝟎 × 𝛉
(i) The distance along any of the small circles (all the parallel of latitudes including
the equator since𝑐𝑜𝑠 0° = 1) where R= 3437 Nm, 𝜃 is the difference in longitudes and
lat is the common circle of latitude is calculated by
𝛉
Distance = 𝟑𝟔𝟎 × 𝟐𝛑𝐑𝐜𝐨𝐬 𝐥𝐚𝐭

= 𝟔𝟎 × 𝛉 𝒄𝒐𝒔 𝒍𝒂𝒕

29 by T.H Musondela
KNOT
A speed of 1 nautical mile per hour is called a knot. It is the unit of speed commonly used by
sailors and aviators. In marine and aerial navigation, speed on the Earth’s surface is
traditionally measured in knots and one knot equals one nautical mile per hour

1 knot  1 nautical mile/hour = 1.852 km/h

WORKING WITH BASIC FORMULAS

Working with and understanding basic formulas and how to arrange them is an important
skill necessary in today’s world, especially for an officer on a modern commercial vessel.
Let’s look at a basic formula to calculate Distance, Time, and Speed.
The simple formula is: D = S x T. Where:
D = Distance (usually expressed in nautical miles)
S = Speed (usually expressed in knots)
T = Time (expressed in hours and/or minutes, or tenths of a minute)

Example 1: Your vessel is traveling at 18 knots and you travel for 2 hours. How far did you
travel?

Distance = Speed x Time


D=SxT
= 18 x 2
= 36 nautical miles (nm)

Example 2 : How long (time) will it take your vessel to reach its destination if you travel 35
nautical miles at a speed of 19 knots?
𝐷 35
𝑇 = 𝑆 = 19

T = 1.84 hours. (How many minutes is 0.84 hours? 0.84 x 60 = 50.4 minutes)
T = 1 hour 51 minutes

30 by T.H Musondela
Example 3: A (41o N, 36o W) and B (41o N, 155o E) are two points on the surface of the
earth.
(a) Calculate the shortest distance along the line of latitude from A to B in nautical miles
(b) A ship sails from A to B via the shortest distance at the speed of 350 knots. How long did
it sail?

Solution
(a) The two place have a common latitude therefore the distance is on that latitude which is a
small circle with radius r = 6370 𝐜𝐨𝐬 𝟒𝟏 nm . The angle θ in the formula below is given by the
sum or difference of the longitude. Since we have different direction 36o W and 155o E, we add
them, that is, θ = 36+155 = 191. Smallest angle is not 191o, but 169o.
𝛉
So Distance = 𝟑𝟔𝟎 × 𝟐𝛑𝐑𝐜𝐨𝐬 𝐥𝐚𝐭 [Remember that 𝟐 × 𝟑.𝟏𝟒𝟐 × 𝟑𝟒𝟑𝟕≅𝟐𝟏𝟔𝟎𝟎]

= 𝟔𝟎 × 𝛉 𝒄𝒐𝒔 𝒍𝒂𝒕
Since the measurement is along
= 169 x 60 cos 41o
a small circle the distance is
= 7652.755143458 reduced by a factor of cos 41o.

= 7653 nm (3sf)
𝑑𝑖𝑠𝑡𝑎𝑛𝑐𝑒 7652.755
(b) Time = 𝑠𝑝𝑒𝑒𝑑
= 350
= 21.8650142857143 h𝑜𝑢𝑟𝑠 = 21 hours 52 minutes

Example 4: The coordinates of two points on the Earth’s surface are given by the coordinates
A (50°N, 120°W) and B (30°S, 120°W). Calculate the time taken for a ship to sail the shortest
distance between these two points at an average speed of 40 knots.

(a) The two place have a common longitude therefore the distance is on that longitude which
is a great circle with radius R= 3437nm. The angle θ is the difference of the latitude. Since
we have different direction 50o N and 30o S, we add them, that is, θ = 50+30 = 80.
𝑑𝑖𝑠𝑡𝑎𝑛𝑐𝑒 60 × 80
Time = 𝑠𝑝𝑒𝑒𝑑
= 40
= 120 hours

31 by T.H Musondela
Example 5: M, R and Z are points on the surface of the earth. M is on latitude 55° N, Z and R
are both on latitude 65°S, M and R are on the same N

longitude 30°E and Z is on longitude 45°W.


55°N
M
[Take R=3437 and 𝜋 = 3.142] 65°
30°E

65°S
Z R

(i) Calculate the difference in latitudes between points R and M. S

(ii) Calculate the shortest distance MR on the surface of the earth in nautical miles.
(iii) Calculate the shortest distance ZR on the surface of the earth to the nearest kilometer.
(iv) The local time at Z is 07 20 hours. Find the time at R

Solutions

(i) Latitudes of R and M are 65°S and 55°N respectively .

∴ Difference in latitudes = 65 +55 = 120° (we add since they are on opposite sides of the
equator)
θ
(ii) Distance = 360 × 2πR (iii) ZR = 95 x 60 cos 65o

= 60 × 12 = 2408.9240919

= 7200 nm = 2410 nm

(ii) Difference in longitudes of Z and R = 30 +65 = 95° .


95
Time difference = 15 = 5 hrs 30 minutes so the time = 07 20 +06 30 =13 50

NOTE: The time at M is same as the time at R since the two points lie on the same
longitude

32 by T.H Musondela
Example 6: A plane flies due south from town A (37.5˚N, 15˚E) to town B (15˚S, 15˚E) in
9hours. Find the speed of the plane in knot.

Solution
The difference in latitude = 37.5˚ + 15 = 52.5˚
The distance AB = 52.5 × 60 = 3150 nm
3150 nm
 The speed  distance   350 knots
time 9h

Example 4
Find the shortest distance from (42°N, 110°E) to: (a) (15°N, 110°E) (b) (18°S, 110°E).

Solution
(a) Angular distance  42   15  (b) Angular distance  18   42 
 27   60 
 27  60  60  60
 1620  3600
Distance  1620 n mile Distance  3600 n mile

FINDING LATITUDES OR LONGITUDES FROM GIVEN SITUATIONS

Example 1: Your vessel leaves Latitude 15° south and travels due north for 45° of latitude.
What is the latitude of arrival?

Solution
If you begin at 15° south and travel north, you can see that you will be heading toward the
Equator. You reach the Equator (0°) after traveling 15° north. However, the question says
you’ve traveled north for 45° of latitude. Out of the total of 45° latitude, we have already
traveled 15° upon reaching the Equator, so we have another 30° north to travel (45° - 15° =
30°). When you travel the remaining 30° north from the Equator (0°), you arrive at 30° N

33 by T.H Musondela
Example 2: Your vessel leaves Longitude 16° east and travels due east for 100° of longitude.
What is the longitude of arrival?

If you begin at 16° east and are traveling another 100° in the same direction, you add 16°
east + 100° east = 116° east. If you look at the map above, you can see Workboat Mate
Program Latitude and Longitude that longitude runs east until 180°. Because our answer is
less than 180°, we can see that 116° east is both practical and correct.

Example 3
A jet flies 1966 nautical miles due west from town P (35⁰N, 25.5⁰ E) to town Q, find
the position of town Q.
Solution
Due west implies that Q is also on latitude 35⁰N. so we first find the difference in longitude 𝜃.
1966
60 × θ cos 35° = 1966, 𝜃 = 60cos 35 = 40.0007140250837 ≈ 40.0
Since the longitude of P is on the eastern hemisphere and the direction of the jet is westward
then longitude of Q =|25.5 - 40.0| = | - 14.5| = 14.5 ° 𝑊 ∴ 𝑸(𝟑𝟓°𝑵, 𝟏𝟒.𝟓°𝑾)

Example 4
A jet flies 1966 nautical miles due East from town P (35⁰N, 25.5⁰ E) to town Q, find
the position of town Q.

Solution

Due East implies that Q is also on latitude 35⁰N. So we first find the difference in longitude
𝜃.

60 × θ cos 35° = 1966


1966
𝜃 = 60cos 35 = 40.0007140250837 ≈ 40.0

Since the longitude of P is on the eastern hemisphere and the direction of the jet is eastward
the longitude of Q =|25.5 + 40.0| = 65.5 ° 𝐸 ∴ 𝑸(𝟑𝟓°𝑵, 𝟔𝟓.𝟓°𝑬)

34 by T.H Musondela
PUPIL’S ACTIVITY

1. A vessel leaves Longitude 45° east and travels due east for 110° of longitude. What is the
longitude of arrival?

2. A vessel leaves Latitude 35° N and travels due south for 50° of latitude. What is the
latitude of arrival?

3. A vessel leaves Longitude 35° W and travels due east for 110°of longitude. What is the
longitude of arrival?

4. A jet flies 3000 nautical miles due west from town A (20o S, 15o E) to town B, find the
position of town B

4. A jet flies 2540 nautical miles due west from town P (30⁰ s, 70⁰ E) to town Q, find the
position of town Q.

5. A jet flies 2540 nautical miles due East from town P (35⁰N, 15⁰ W) to town Q, find the
position of town Q.

6. A jet flies 3000 nautical miles due west from town A (20o S, 15o E) to town B, find the
position of town B

7. P and Q are two points on the Earth’s surface with coordinates (27°N, 30°W) and (39°N,
30°W) respectively.

(a) Calculate the distance between P and Q in nautical miles.

(b) Use 1 M =1.852 km to give the distance, PQ, to the nearest kilometre.

8. An aircraft leaves Newtown (15⁰ N, 36⁰E) and flies due east to fort Jameson (15⁰N, 102⁰E)
at an average speed of 250 knots. Calculate the time taken.

9 . A jet flies 2540 nautical miles due west from town P (30⁰ s, 70⁰ E) to town Q, find the Q

10. A jet flies 1966 nautical miles due west from town P (35⁰N, 25.5⁰ E) to town Q, find the
position of town Q.
11. A jet flies 1966 nautical miles due East from town P (35⁰N, 25.5⁰ E) to town Q, find the
position of town Q.
12 . A jet flies 3000 nautical miles due west from town A (20o S, 15o E) to town B, find the
position of town B

35 by T.H Musondela
DISTANCE OVER THE POLES
Meridians with longitudes x° E (or W) and (180° - x) °W (or E) form a great circle through
both poles. If the longitudes of A and B are supplementary (sum to 180) then points A and B
are diametrically opposite. In this case the shortest distance between the two points A and B
is via either the North Pole or South Pole.
Example 1

The points A (50°N, 30° E) and B (50° N, 150 °W) are two points on the surface of the earth.
50 °N
B A

150 °W 0°

(a) Find the shortest distance from A to B in nautical miles.


(b) Find the distance between A and B along a parallel of latitude in Nautical miles (nm

SOLUTION
(a) Since the longitudes of A and B are supplementary then the shortest distance is via the North
Pole
N 𝜃=180 –(50+50) = 80 or 𝜃=180 –(50+50) = 80
B A ∴ 𝐴𝐵 = 60 × 80 80
𝐴𝐵 = × 2 × 3.142 × 34 37
150° W 30° E 360
50° 𝜃° 50° = 4800𝑛𝑚
= 4799.579555555556
= 4800𝑛𝑚

(b) In this case the 𝜃=30+150 = 180. Both points lie on latitude 50° N
𝜃=30° + 150 = 180 or 𝜃=30° + 150 = 180
∴ 𝐴𝐵 = 60 × 180cos 50 180
𝐴𝐵 = × 2 × 3.142 × 3437cos (50)
= 6942.1061846146249 360

= 6940 𝑛𝑚 = 6941.4981075358619
= 6940 𝑛𝑚

36 by T.H Musondela
Example 2
The diagram below shows a wire model of the earth, The circle of latitude in the north is 80
°𝑁 and the circle of latitude 0°. The meridian 𝑁𝑃𝑅𝑆 is 60°𝐸 and meridian 𝑁𝑄𝑇𝑆 is directly
opposite𝑁𝑃𝑅𝑆. N
Q
80° N
P
T

R
60° E

(a) State the position of the point Q.


S
(b) Find the distance along the circle of latitude 80°𝑁 between Q and 𝑃 in 𝑘𝑚.
(c) Calculate the shortest distance between 𝑃 and 𝑄
(d) If the time at 𝑅 is 20 00ℎ𝑟𝑠, what is the time at 𝑇

Solutions
(a) The meridian 𝑁𝑃𝑅𝑆 is 60°𝐸 and meridian 𝑁𝑄𝑇𝑆 is directly opposite𝑁𝑃𝑅𝑆. So Q is
on longitude (180 ‒ 60)°𝑊 = 120 𝑤 , ∴ 𝑄(80𝑁, 120𝑊)
180
(b) QP = 360
× 2 × 3.142 × 6370 × 𝑐𝑜𝑠 80

= 3475.5𝑘𝑚
= 𝟑𝟒𝟖𝟎𝒌𝒎 (3 𝑠.𝑓)
𝜃
(c) = 360 × 2𝜋𝑅 , 𝜃=180 –(80+80) = 20
20
= × 2 × 3.142 × 6370
360
𝐷 = 2223.8377778 𝑘𝑚
𝐷 = 𝟐𝟐𝟐𝟎𝒌𝒎(3 𝑠.𝑓)
180
(d)Time differed = 15
= 12 ℎ𝑜𝑢𝑟𝑠

∴ time at 𝑇 = 20 00 ‒ 12 00 = 𝟎𝟖 𝟎𝟎 𝒉𝒐𝒖𝒓𝒔

37 by T.H Musondela
Example. 3
Cherepovets, Russia is located approximately at (60°N, 38°E) and Mt. Logan, Canada is
located approximately at (60°N, 142°W).

a) How far apart are the two locations, measured along the parallel of latitude 60°N°

b) Calculate the great circle distance between Cherepovets and Mt. Logan.

Solution
(a) Both Cherepovets and Mt. Logan are on latitude 60°N and they are 38° + 142° = 180°
apart. This means that on latitude 60°N they are one half the circumference of this small
circle apart. We know that 𝑟 = 𝑅𝑐𝑜𝑠 60° and hence the distance along the parallel of
latitude 60°N is
𝜽
D = 𝟑𝟔𝟎 × 𝟐𝝅𝒓 𝜽
D = 𝟑𝟔𝟎 × 𝟐𝝅𝑹
𝜽
= 𝟑𝟔𝟎 × 𝟐𝝅𝑹 𝒄𝒐𝒔 𝒍𝒂𝒕 𝟏𝟖𝟎
𝟏𝟖𝟎
= 𝟑𝟔𝟎
× 𝟐 × 𝟑.𝟏𝟒𝟐 × 𝟑𝟒𝟑𝟕
= 𝟑𝟔𝟎 × 𝟐 × 𝟑.𝟏𝟒𝟐 × 𝟔𝟑𝟕𝟎 𝒄𝒐𝒔 𝟔𝟎°
= 𝟏/𝟔 × 21598.108
= 𝟎.𝟓 × 𝟒𝟎𝟎𝟐𝟗.𝟎𝟖 × 𝒄𝒐𝒔 𝟔𝟎°
= 𝟏𝟎 𝟎𝟎𝟕.𝟐𝟕 𝒌𝒎.
= 3599.7𝒏𝒎. (1dp)

b) But since their longitudes are 180° apart, both Cherepovets and Mt. Logan are located
on a great circle through the North and South Poles.
The sector angle is 180° - (2 × 60°) = 60°.
Kilometers nautical mile

𝜽 𝜽
D = 𝟑𝟔𝟎 × 𝟐𝝅𝑹 D = 𝟑𝟔𝟎 × 𝟐𝝅𝒓
𝟔𝟎 𝜽
= 𝟑𝟔𝟎 × 𝟐 × 𝟑.𝟏𝟒𝟐 × 𝟔𝟑𝟕𝟎 = 𝟑𝟔𝟎 × 𝟐𝝅𝑹 𝒄𝒐𝒔 𝒍𝒂𝒕
𝟏 𝟏𝟖𝟎
= 𝟔 × 𝟒𝟎𝟎𝟐𝟗.𝟎𝟖 = 𝟑𝟔𝟎
× 𝟐 × 𝟑.𝟏𝟒𝟐 × 𝟑𝟒𝟑𝟕 𝒄𝒐𝒔 𝟔𝟎°

= 6671.5 𝒌𝒎. (1dp) = 𝟎.𝟓 × 21598.108 × 𝒄𝒐𝒔 𝟔𝟎°


= 5399.527 𝒏𝒎.

38 by T.H Musondela
Example.4
P and Q are two places on the surface of the Earth both in latitude 65N. The longitude of P is
33°W and PQ is the diameter of the circle of latitude 65° N.
N
𝑄
65°N
33°W
𝑃

(a) Calculate the longitude of Q. S


(b)An aircraft flew at a speed of 600 knots from P to Q along the shortest route over the North
Pole.
(i) Calculate the distance, in nautical miles, flown by the aircraft
(ii) Calculate the time taken for the aircraft to reach Q,
(iii) Calculate the distance, in nautical miles, between P and Q measured along their common
circle of latitude
(c)A second aircraft flew from P to Q along the circle of latitude 65° N and departed 30
minutes after the first aircraft. Find the speed at which the second aircraft had to travel in
order that both aircraft arrived at Q at the same time.

Solutions
(a) Longitude of Q = (180- 33) E = 147 E

(Note that PQ is the diameter so the two longitude must add up to 180)
(b) (i) PQ via N= 60 but  = 180 – (65+ 65) = 50. ∴ PQ = 60  50 3000Nm
𝐷𝑖𝑠𝑡𝑎𝑛𝑐𝑒 3000
(ii) TIME = 𝑆𝑝𝑒𝑒𝑑
= 600
= 5 hours

(iii) PQ via common latitude = 60 cos lat but  = 33+147=180. ∴ PQ = 60 180 cos65 
4564.2772 Nm  4564.3Nm (1dp)
(c) Distance along the circle of latitude 65° = 4564.3Nm and time = 5 hrs - 30 min = 4.5 hrs
𝑑 4564.2727
S=𝑡= 4.5
=1014.28 knots

39 by T.H Musondela
Example 3: An aircraft leaves X (70° N, 150° W) at 12 00 hours and flies due North at
constant speed. After flying over the north pole, it passes directly over P (60° N, 30° E) at
1930 and continues to fly due south at the same constant speed.
(a) Calculate the length, in nautical miles, of that part of the aircraft’s flight between X and P.
(b) Calculate the speed, in knots, of the aircraft.
(c) A radar tracking station at P can detect aircraft within a range of 480 nautical miles.
Find the number of minutes during which the aircraft can be detected by the tracking station.
(d) A second tracking station at Q (60° N, 5° E) can also detect aircraft within a range of 480
nautical miles. A second aircraft is flying due west along the parallel of latitude 60° N.
(i) Calculate the longitude of this aircraft when it is first detected by the station at Q
(ii) Calculate the distance, in nautical miles, traveled by this aircraft while it is within the
contact of both radar tracking stations.

Solutions

(a) 𝜃 =180 – (70 + 60 ) = 50º ∴ distance flown is 60 × 50 = 3000 nautical miles.

d 3000
( b) Average speed = = = 400 knots
t 7.5

d 480  480
(c) time = =  60 = 144 minutes
s 400

480
(d) (i) Longitude = 5ºE + = 21ºE
60 cos 60

(ii) 480 – D + 480 – D + D = 60 × 25 cos60


960 – D = 750 nm
D = 960 – 7 50 nm
D = 210 nautical mile

40 by T.H Musondela
EXERCISE 1
1. An aircraft can fly from X (65° N, 100° W) to Y (65° N, 80° E) along two different routes.
(a) Find the distance in km over the North pole
(b) Find the distance in km a long the line of latitude

2 Point V is on latitude 20° S and longitude 130° E while point T is on latitude 20° S and
longitude 50°E. Taking π to be 3.142 and R to be 6370km,
( a ) Calculate the distance between V and T along the circle of latitude
(b) Calculate the distance between V and T via the south pole.

3. Given that the earth is a sphere of radius 6370km and  = 3.14, calculate

(i) the length of the equator

(ii) the length of the meridian from the North Pole to the South Pole through Greenwich.

4. The position of two towns X and Y are given to the nearest degree as X (45° N, 110° W)
and Y (45° N, 70° E). Take = 3.142, R = 6370km.
(a) Find the distance between the two towns along the parallel of latitude in km.
(b) Find the distance between the towns in nautical miles.
(c) A plane flew from X to Y taking the shortest distance possible. It took the plane 15hrs to
move from X and Y. Calculate its speed in Knots.
(d) Find the local time at Y when the local time at X on 10th April is 10.00pm.

5. (a) Find the shortest distance in km over the North Pole from point P (65° N, 12 °W) to
point Q (50 °N, 168 ° E)
(b). A and B both lie in latitude x° N, their longitudes are 126° w and 54° E respectively.
The shortest distance on the surface of the earth between them is 3480 nautical miles.
(i) Find the value of X
(ii) The distance between A and B along a circle of latitude x.

41 by T.H Musondela
EXERCISE 2
1. (a) The latitude and longitude of two stations P and Q are (47 o N, 25 o W) and (47 o N, 70 o
W) respectively. Calculate the distance in nautical miles between P and Q along the latitude
(b) Find the distance in nautical miles and kilometres between two points A(60 o N, 45 o E)
and B(60 o N, 60 o W)
(c) Cargo ship leaves a town, which is at (41 o N, 53 o W) and sails due east for 32hours to a
point (41 o N, 37 o W) Calculate its average speed in knots
(d) Cargo ship leaves a town, which is at (27 o N, 178 o W) and sails due west for 8 hours to
a point (27 o N, 160 o E) Calculate its average speed in knots

2. A (10°N, 140°E), B (29°S, 140°E) and C (29°S, 149°E) are three points on the surface of
the Earth. An aircraft flies due south from A to B in 5 hours 30 minutes.
(a) (i) Calculate the distance, in nautical miles, that it flies,
(ii)Calculate its average speed, in knots,
(iii) Calculate the further distance, in nautical miles, that it flies if it then travels due east
from B to C.
(b) Another aircraft flies at an average speed of 150 knots and has fuel for 3 hours’ flying
time. The pilot is instructed to fly due west from B and then return to B along the same route
without refueling. Calculate, in degrees and minutes, the longitude of the most westerly point
that he can reach before he must turn back.

3. Consider points A (50°N, 29°E) and B (50°N, 149°W) .Take 22⁄7 and radius of the earth
R = 6370 km. Find:
(a) (i) The distance between A and B along a parallel of latitude in Kilometres (km)
(ii) The distance between A and B along a parallel of latitude I Nautical miles (nm)
(b) The shortest distance from A to B in nautical miles.
(c) An aircraft takes 54 hours to fly between the two towns A and B along the great circle.
Calculate its speed in knots correct to 2 significant figures.

42 by T.H Musondela
EXERCISE 3
1. Find the shortest distance in km over the North or South Pole between each pair of points
(Radius of Earth 6370 km, π = 3.142)
(a) A (70°N, 50°W) to B (60°N, 130°E) (b) C (84°S, 17°E) to D (75°S, 163°W)
(c) P (10°S, 119°W) to Q (00, 61°E) (d) X (50°N, 10°W) to Y (66°N, 170°E)

2. Find the shortest distance in nautical miles between each pair of points:
(a) 10°N, 41°W) to (27°N, 41°W) (b) (110S, 165°E) to (85°S, 165°E)
(c) (80°N, 100°W) to (70°N, 80°E) (d) (250N, 840W) to (250N, 600W)

3.(i) Find the distance in km between the following pairs of points on the equator:
(a) M (0° , 65°W) to N (0°, 87°W) (b) P (00, 117°E) to Q (00,94.6°E)
(ii) Find the shortest distance in km along a line of latitude between each pair of points
(a) (40°N, 75°W) to (40°N, 15°W) (b) (25°N, 102°E) to (25°N, 17°E)
(c) (17°S, 11°W) to (17°S, 25°E) (d) (63.5° S, 90°) to (63.5°S, 0°)

4. An aircraft files due North or South from each of the following points. Find its new
position.
(a) 60 nm South from (70°N, 10°W) (b) 162 nm Miles North from (11°S, 111°W)
(c) 40 nm north from (17°N, 10°W) (c) 1966 nm north from (17°N, 10°W)

5. (i) Find the distance in nautical miles between each pair of points
(a) A (60° N, 30°W) to B (10° S,30°W) (b) P (0° N, 35° W) to Q (10° 25S, 35° W)
(c) X (14.5° N, 160° E) to Y (20° S, 160° E) (d) C (72.6° N, 100° W) to D (61.1° N, 100° W)
(ii) . An aircraft flies west from (0°, 10°E) to arrive at (0°, 17°W) in 4 hours. Find its speed in
km/h (Take R = 6370 KM, π = 3.142)
(ii) A ship sails due West from (0°, 17°W) at a speed of 20 knots. Find its position after 3
hours

43 by T.H Musondela
EXERCISE 4
1 . An aircraft flies due South from A (65°N, 10°W) at a speed of 50° km/h .Find its position
after 7 hours
2. A frigate sails due North from (19°S, 112°W) at a speed of 30 knots. Find its position after
3 hours.
3. Calculate the distance between Utopia (30º S, 12º E ) and Nowhere ( 70ºN, 12ºE )
22
measured on the surface of the earth. Take π to be 7
and the radius of the earth to be 6370.

4. Calculate the distance between Utopia ( 30º S, 12ºE ) and Nowhere ( 42ºN, 12ºE )
22
measured on the surface of the earth. Take π to be 7
and the radius of the earth to be 6370.

5. Mogadishu and Kinshasa are on longitudes 45°𝐸 and15°𝐸. Find the time in Kinshasa when
it is 11 30 hours in Mogadishu.
6. A soccer match kicked of at 14 30 hours at A (33°𝑁,30° 𝑆). What would be the time at
town B (12° 𝑆,32°𝐸)
7. Two points P and Q are on the same longitude 33°𝐸. Given that P is on latitude 70°𝑁 and
that PQ is 7200nm, find the position of Q.
8. A jet flies due south from town A (33°𝑁,32°𝐸) town B (12° 𝑆,32°𝐸)in four 4 hours. Find
22
the average speed of the jet in Km/h. Take 𝜋 = 7
and R = 6370 km.

9. Find the shortest distance over the earth’s surface between (32°𝑁,12°𝐸) town B (40° 𝑁
22
,168°𝐸) . Take 𝜋 = 7
and R = 6370 km.

10 . Find the shortest distance over the earth’s surface between (67°𝑆,40°𝐸) town B (23° 𝑁
22
,40°𝐸) . Take 𝜋 = 7
and R = 6370 km.

11. Find the distance (in km) between towns P(12.4°S, 30.5°E) and Q (12.4°S, 39.8°E) along
a line of latitude, correctly to 4 decimal places.
12. A jet flies from (34°N, 12°E) to (34°E , 24°E) in 1.5 hrs. Find its average speed in knots
P and Q are two points on a geographical globe of diameter 50 cm.
13. Calculate the distance between two places A (30°𝑺,𝟏𝟐°𝑬) and (60°𝑵,𝟏𝟐°𝑬). (Take the
22
radius of the earth to be 6370km and 𝜋 as 7

44 by T.H Musondela
Exercise 5
1. An aircraft starts from A(0°, 15°E) and flies 1500 nautical miles due east along the equator
to B. Find the longitude of B.
2. Another aircraft flies due north from an airfield P(18°S, 31°E) to Q(30°N, 31°E).
Find the distance in nautical miles which it flies.
3. A third aircraft flies 4200 nautical miles along the circle of latitude 18°S from the airfield
P to R. Find the two possible longitudes of R.
4. Two points X and Y lie on the same circle of latitude. Measured eastwards along the circle
of latitude, the distance from X and Y is twice the distance from Y to X. Given that the
longitude of X is 50°E, find the longitude of Y.
5. Calculate the following circles of latitude: (a) 60°N (b) 31°N (c) 78.2°S
6. Find the distance in nautical miles along the equator between A (0°, 65°W) and B (0°,
10°W)
7. Find the distance in nautical miles along the line of latitude between C (30°N, 65°W) and B
(30° N, 10° W).
8. Find the speed in km/h of an aircraft which travels between the following points in the time
given:
(a) (590N, 100W) due East to (59°N, 31°E) in 2 hours.
(b) (0°N, 170°W) due East to (0°N, 0°W) in 8 hours.
9 .Oslo lies on the circle of latitude 60°N. Given that the radius of the earth is 6370km and
Cos 60° =0.5, Calculate the circumference of the circle of latitude on which Oslo lies correct
to the nearest km. (Take π to be 3.142)
10. An aircraft can fly from X (65°N, 100°W) to Y (65°N, 80°E) along two different routes.
(a) Find the distance in km over the North Pole.
(b) Find the distance in km along the line of latitude.
11(i) Find the distance in km between M (9°S, 33°E) and B (65° N, 33°E) on the surface of
the earth.
(ii) An aeroplane takes off from T (5°S, 33°E) to T (5°S, 39°E) at a speed of 332km/h. If it
leaves T at 3:00 p.m., at what time will it arrive at Ta airport?
45 by T.H Musondela
Exercise 6
1 . A (10°N, 140°E), B (29°S, 140°E) and C (29°S, 149°E) are three points on the surface of
the Earth. An aircraft flies due south from A to B in 5 hours 30 minutes.
(i) Calculate the distance, in nautical miles, that it flies,
(ii) Calculate its average speed, in knots,
(iii) Calculate the further distance, in nautical miles, that it flies if it then travels due east from
B to C .
(iv) Another aircraft flies at an average speed of 150 knots and has fuel for three hours’ flying
time. The pilot is instructed to fly due west from B and then return to B along the same route
without refueling. Calculate, in degrees and minutes, the longitude of the most westerly point
that he can reach before he must turn back.

2 . A ship leaves an island ( 5° N, 45° E) and sails due east for 120 hours to another island.
The average speed of the ship is 27 knots.
(a) Calculate the distance between the two islands
(i) in nautical miles (ii) in kilometers
(b) Calculate the speed of the ship in kilometers per hour
(c) Find the position of the second island
(take 1 nautical mile to be 1.853 Km and the radius of the earth to be 6370 Km)

3. A certain parallel of north latitude has a circumference of 33 180km.


(a) Calculate the radius of this parallel of latitudes.
(b) Assuming the radius of the earth to 6370km, calculate also the latitude of all points on
this parallel.
8 If the distance between two points A and B measured along this parallel of latitude is
4424km, (a) calculate the difference in longitude between the points A and B.
(a) If the longitude of A is 100E, state two possible values for the longitude of B.

8. Point P (36° N, 170°W) and point Q (36° N, 150E °) are on the surface of the earth.
Calculate the shortest distance along a circle of latitude between the two points

46 by T.H Musondela
EXERCISE 7
1. A (50°S, 20°W), B (50°S, 60°W) and C (15°N, 60°W) are three points on Earth’s surface.
The radius of Earth is approximately 6370 km. Take 𝜋 to be 3.142 where necessary.
(i) Neatly sketch the above information
(ii) Calculate the local time at A if it is 9 25 am in B.
(iii) Calculate the radius of the parallel of latitude 50°S to the nearest kilometre.
(iv) Calculate the shortest distance between A and B along the parallel of latitude 50°S.
(v) An SOS message is sent from C to B. A jet flies from B to C along longitude 60 °W at the
speed of 750 km/h to answer the SOS message. Calculate the time it takes to fly from B to C

2. Two towns on the earth’s surface are located at P (07°N,30°E) and Q (13°S,30°E). A pilot
plans to fly from P to Q the shortest route between the two towns.
(a) Calculate the shortest distance between P and Q in km.
(b) Find the distance in nautical miles (nm)
(c) The speed of the aircraft is 360 knots. Determine how long it takes to fly from P to Q.

3. The position of two towns are A (30°S, 20°W) and B (30°S, 80°E) find
(a) the difference in longitude between the two towns.
(b) the distance between A and B along parallel of latitude in km (take radius of the earth as
6370km and π=22/7
(c) the distance between A and B along parallel of latitude in km in nm.
(d) Find local time in town B when it is 1.45pm in town A.

4. The position of two towns P and Q are given to the nearest degrees as P(45°N, 110°W) and
Q (45°N, 70°E) Take = 3.142 and Radius of the earth R = 6370km. Find
(a) The distance between the two towns along the parallel of latitude in km.
(b) The distance between the towns along a parallel of latitude in nautical miles.
(c) A plane flew from P to Q taking the shortest distance possible. It took the plane 15 hours
to move from P and Q. Calculate it’s speed in knots

47 by T.H Musondela
EXERCISE 8
7. The position of two towns A and B on earth surface are (36°N, 49°E) and (36°N, 131°W)
respectively. Take R = 6370
a) Find the difference in longitude between the town A and B.
b) Calculate the distance between A and B along the latitude in
(i) Nautical miles (ii) kilometres
(c) Another town C is 840km due east of town B and on the same latitude as town A and B.
(i) Find the position of town C.
(ii) If the local time in B is 0730 find the local time in C.

8. A plane S flies from a point P (40°N, 45°W) to a point Q(35°W, 45°W) and then onto a
point T (35°N, 135°E).
a) Given that the radius of the earth is 6370km, find the distance P to Q in km.
b) Find in nm;
i) the shortest distance between Q and T.
ii) the longest distance between Q and T (to the nearest tens).
c) Find the difference in time taken when S flies along the shortest and longest routes if its
speed is 420 knots.

18. A ship leaves an island (05°N, 45°E) and sails due east for 120 hours to another island.
The average speed of the ship is 27 knots.
(a) Calculate the distance between the two islands
(i) in nautical miles (ii) in kilometers
(b) Calculate the speed of the ship in kilometers per hour
(c) Find the position of the second island (take 1 nautical mile to be 1.853 Km and the radius
of the earth to be 6370 Km)

21. A point M (60 °N, 18 °E) is on the surface of the earth. Another point N is situated at a
distance of 630 nautical miles east of M.
(a) Find the longitude difference between M and N;
(b) Find the position of N.

48 by T.H Musondela
EXERCISE 9
1. A plane leaves an airport A (38.5°S, 37.5°W) and flies dues North to a point B on latitude
520N.
(a) Find the distance covered by the plane
(b) The plane then flies due east to a point C, 2400km from B. Determine the position of C
Take the value Π of as 22/7 and radius of the earth as 6370 km

2. The position of two towns X and Y are given to the nearest degree as X (45° N, 10°W) and
Y (45° N, 70°W)
(a) Find the distance between the two towns in
(i) Kilometers (take the radius of the earth as 6371)
(ii) Nautical miles (take I nautical mile to be 1.85 km)
(c) Find the local time at X when the local time at Y is 2.00 pm.

3. A tourist took 1 h 20 minutes to travel by an aircraft from town T(3°S, 35°E) to town
U(9°N, 35°E). (Take the radius of the earth to be 6370km and π=22/7
(a) Find the average speed of the aircraft.
(b) After staying at town U for 30 minutes, the tourist took a second aircraft to town V
(9°N, 5°E). The average speed of the second aircraft was 90% that of the first aircraft.
Determine the time, to the nearest minute; the aircraft took to travel from U to V
(c) When the journey started at town T, the local time was 0700h. Find the local time at V
when the tourist arrived.

4. Nakano, Japan (37°N, 138°E) and Adelaide (35°S, 138°E) are two places on the surface of
the earth.
(a) Find the distance between Nakano, Japan (37°N, 138°E) and Adelaide (35°S, 138°E)
(b) A jet due south from Nakano, Japan (37°N, 138°E) to Adelaide (35°S, 138°E) at the
speed of 350 knots. Find how long it took flying.

49 by T.H Musondela
EXERCISE 10
1. The position of two satellite station A and B on the earth’s surface are (36 °N, 50°E) and
(36° N, 130°W) respectively. ( use = 3.142 and R = 6370 KM)
a) Find the distance along the small circle in km.
b) Find the shortest distance between A and B IN nm
c) If time A is 1700hrs, calculate the time at B in 12 hrs system.

2. The position of the two towns P and Q on the earth’s surface are (60°N 139°E) and (60°N,
41°W) respectively. Take the radius of the earth as 6370km,
a) Find the latitude difference between P and Q.
(b) i) calculate the distance between P and Q (i) via the North pole.
(ii) Along the parallel of latitude.
(c) Another town R is 420km east of town P and on the same latitude as P and Q. Find the
longitude of town R.

3. The positions o two towns A and B on earth’s surface are (60°N, 139°E) and (60°N, 41°W)
respectively
a) Find the difference in longitude between A and B
b) Given that the radius of the earth is 6370km, calculate the distance between A and B in km
c) Another town C is 420 km due East of town B and on the same latitude A and B find the
longitude of town C

4. (a) Point P (40°S, 45°E) and point Q (40°S, 60°W) are on the surface of the Earth.
Calculate the shortest distance along a circle of latitude between the two
(i) in kilometers
(ii) in nautical mile
(b) Points A and B lies on the same circle of latitude P0N if A and B are on longitude 41°W
and 3°Erespectively and the distance between them is 1370nm. Calculate the latitude P.

50 by T.H Musondela
EXERCISE 11
1. Two points on the surface of the earth are A (40°N, 30°W) and B (20°S, 30°W). Given that
the radius of the earth is 6370km, determination the shortest distance between the two points.
(Take 22/7)

2. Two points A and B are found on the earth's surface. The position of A is (52°S, 66°W)
and B (52°S, 114°E). Use Earth’s radius as 6 370 km.
(a) Find the longitude difference between A and B.
(b) Calculate the shortest distance between A and B along:
(i) Find the latitude in kilometres to the nearest whole number.
(ii) Find the longitude in kilometres to the nearest whole number.
(c) A plane travelling at 800 km/h leaves point A at 10 00 and flies through South Pole to
point B. Find the local time the plane arrives at point B to the nearest minutes.

3. The position of two A and B on the earth’s surface are ( 36° N, 49°E) and (36°N, 131° W)
respectively. Take the radius of the earth is 6370,
(a) Find the difference in longitude between town A and town B
(b) Calculate the distance between town A and town B.
(c) Another town, C is 840 due east of town B and on the same latitude as towns A and B.
Find the longitude of town C.

12. Two towns A and B lie on the same latitude in the northern hemisphere. When its 08 00 at
A, the time at B is 1100 .
a) Given that the longitude of A is 150 E find the longitude of B.
b) A plane leaves A for B and takes 31/2 hours to arrive at B traveling along a parallel of
latitude at 850km/h. Find:
(i) The radius of the circle of latitude on which towns A and B lie.
(ii) The latitude of the two towns (take radius of the earth to be 6371km)

51 by T.H Musondela
EXERCISE 12
1. Calculate the shortest distance between X(40°N,80°W) and Y (40°N,100°E) in kilometers
taking π =22/7 and Radius = 6371km. (Give your answer to the nearest whole number)

2. The latitude and longitude of two stations P and Q are (47°N, 25°W) and (47°N, 70°W)
respectively. Calculate the distance in nautical miles between P and Q along the latitude 47°N

3 . A globe representing the earth has a radius of 0.2m. Points P (60°N, 140°E) and Q (60°N,
120°W) are marked on the globe. If O is the centre of the latitude 60°N, find the area of the
minor sector

4 . A plane flies from point P (40°N, 50°E) towards West to a point Q. Given that the plane
covers a distance of 10,000km what is the position of Q. (Take = 22/7, radius of the earth
6370km)

4. Points P(30°N, 20°W), Q(30°N, 40°E), R(600N, a°E) and S(b°N, c°W) are four points on
the surface of the earth. R is due North of Q and S is due West of R and due North of P.
(a) State the values of a, b and c.
(b) Given that all distances are measured along parallels of latitudes or along meridians, and
in nautical miles, find the distance of R from P using two alternative routes via Q and S.
(c) Two pilots start flying from P to R one along the route PQR at 400 knots and the other
along PSR at 300 knots which one reaches R earlier and by how long?

5. A jet on a rescue mission left town A(35°S, 15°E) to town B(45°N, 15°E) and then to town
C(45°N, 45°W). If 1o subtends 60nm and the radius of the earth is 6370km. Find;
(a) the distance in nautical miles from A to C via B correct to 4 s.f
(b) the distance in kilometers from A to B to the nearest km
(c) the jet flew at 840km/h from A to C. If the jet left town A at 8.15a.m, what time will it
arrive at town C in local time

52 by T.H Musondela
EXERCISE 13
1. An aircraft flies from a point A (1°15’S, 37°E) to a point B directly North of A. The arc
AB subtends an angle of 4890 at the centre of the earth. From B the aero plane flies due west
to a point C on longitude 230W. Take radius of the earth as 6370km.
(a) (i) State the location of B
(ii) Find the distance in km traveled by the aero plane between B and C
(b) (i) The aeroplane left B at 1.00am local time. What was the local tie at C?
(ii) If it maintained an average speed of 840km/h between B and C, at what local time did it
arrive at C?

2 . A jet flies from town X (50°S, 20°E) directly to Y(50°S, 28°W) and then due South for
1200 nautical m to Z
(a) (i) Find the latitude of Z
(ii) Calculate the distance XY along a parallel of latitude 50°S in km
(b) (i) Given that the average speed of the jet is 400 knots, calculate the time taken to reach Z
from X to the nearest 0.1hour
(ii) Find the time of arrival at Z given that the plane left X at 7.40a.m. Take π = 22/7 and
radius of the earth to be 6370km

3. P, Q and R are points on the surface of the earth such that P (60ºN, 20ºW), Q (60°S,
20°W) and R(60°N, 80°E) find:
a) The shortest distance between P and Q on the surface of the earth in kilometres and
nautical miles(nm)
b) The length of latitude 60ºN and hence the length of the minor arc PR in kilometres
c) The distance from P to the North Pole

9. Two points A (70°N, 15°E) and B lie on the same circle of latitude on the earth’s surface.
Given that the shortest distance between the two points along the circle of latitude is
2133.6km. Giving coordinates to the nearest degree, find the location of B. (Take π = 22/7
and radius of earth = 6370km)

53 by T.H Musondela
EXERCISE 14
1. Find the distance between Perth (32°S, 116°E) and Beijing (40°N , 116°E).
2. What is the distance from Quebec (47°N, 71°W) to Santiago, Chile (34°S, 71°W)?
How long would it take to fly at 600 km/h between these cities?
3. What is the distance between the Indian cities Aurangabad (20°N, 75°E) and Puri (20°N,
86°E) along the parallel of latitude?
3. What is the distance between Montevideo (35°S, 56°W) and Canberra (35°S , 149°E) along
the parallel of latitude?
4. Find the time taken for a crested tem to fly from Flinders Island (40°S, 148°E) to King
Island (40°S, 144°E) if it flies at 10 km/h along the parallel of latitude.
5. A plane flying at 400 km/h travelled directly north for 6 hours and 40 minutes before
making an emergency landing. If the plane took off from Hobart (43°S, 147°E), where did it
make the landing?
6. A balloon floats at an average speed of 12 km/h on the prevailing breezes from Bordeaux
(45°N, 1°W) to Krasnodar, Russia (45°N, 39°E). If it was released at 8 00 hours, what was
the time in Bordeaux when it came down in Russia?
7. .Find the distance along the parallel of latitude between Bowen (20°S, 148°E) and Port
Hedland (20°S, 119°E).
8. A pane leaves an airport P (10°S, 60°E) and flies due north at 800km/hr. By taking radius
of the earth to be 6370 km and 1 nautical mile to be 1.853km,
(a) Find its position after 2hrs
(b) The plane turns and flies at the same speed due West to reach Q longitude 12°W.
Find the distance it has traveled due in West nautical miles
(c) Find the time it has taken
(d) If the local time at P was 1300hrs when it reached Q. Find the local time at Q when it
landed at Q
9. The Tropic of Capricorn is the 23.5°S parallel of latitude.
(a) Calculate how many kilometres this circle is from the North Pole
(b) Calculate how many kilometres this circle is from the Antarctic Circle

54 by T.H Musondela
EXERCISE 15
1. Two points P and Q are found on the earth’s surface the position of P is (52 oS, 66 o W) and
Q (52 o S, 144 o E). Taking earth’s radius as 6370km,
(a) Find the difference in longitude between the two points P and Q
(b) (i) calculate the shortest distance between points P and Q along
(i) the common circle of latitude in km
(ii) The longitude in Km
(c) A plane travelling at 800km/hr leaves point P at 10 00 and sails through South Pole to
point Q. find the local time the plane arrives at point Q to the nearest minute.

2. A (50oS, 20oW) and B (50o S, 60oW) are two points on Earth’s surface. The radius of
Earth is approximately 6370 km.
(a) Neatly sketch the above information;
(b) Calculate the local time at A if it is 9 00 in B.
(c) Calculate the radius of the parallel of latitude 50 o S to the nearest kilometre.
(d) Calculate the shortest distance between A and B along the parallel of latitude 50 o S.

3. Beijing and Perth have coordinates (40ºN, 116º E ) and (32ºS, 116º E ) respectively
(i) What great circle joins Beijing and Perth?
(ii) What is the angular distance between these two cities?
(iii) Hence calculate the shortest distance between Beijing and Perth, to the nearest kilometre,
given that the earth's radius is 6400 km.
(iv) If it is 07 00 in Beijing, what would be the time in Perth?

4. A globe representing the earth has a radius of 35cm. Point A (0°, 10°W), B (0°, 35°E),
P(60°N, 110°E) and Q(60°N, 120°W) are marked on globe.
(a) Find the length of arc AB leaving your answer in terms of T
(b) If O is the center of latitude 60°N, find the area of he minor sector OPR.
(c) If the local time at Q is 10 30 on Monday, determine the local time and day at P

55 by T.H Musondela
EXERCISE 16
1. For this question take 𝜋 = 3.142 and R =6 370 km. where R is the radius of the earth,
(i) Calculate, to the nearest kilometer the length of the circle of latitude 37 °N.
(ii) Two towns, A and B. have coordinates (37°N, 50° W) and (37° N, x° E) respectively. The
distance Town A to B measured along their common circle of latitude is 5390 km, calculate
the value of x.

3. P ( lat 60°N, long 15°E), Q ( lat 60°N, long 48°W), and R (lat 38°S, long 48°W) are three
places on the earth surface. Calculate correct to three significant figures.
(a) The distance PQ measured along the latitude
(b) The distance QR measured along the meridian.
(c) The time taken by an aircraft to cover the distance PQ and QR at an average speed of 800
km/h. ( Take π = 3.142 and the radius of the earth = 6370 km).

4. A yacht sails between two islands in the Pacific Ocean: Norfolk Island (29°S, 168°E) and
Vanuatu (17°S, 168°E).
(a) What is the direction of Vanuatu from Norfolk Island?
(b) Calculate the length of the yacht’s journey in nautical miles.
(c) How long will the journey take if the yacht is sailing at an average speed of 8 knots?

5. A plane flew due north from Hobart at a speed of 330 knots for 5 hours.
(a) How far did it travel in nautical miles?
(b) How far did it travel in kilometres?
(c) What is its current position if the coordinates of Hobart are (43°S, 147°E)?

3. Beijing, China and Perth have coordinates (40°N, 116°E) and (32°S, 116°E) respectively.
(a) What great circle joins Beijing and Perth?
(b) What is the angular distance between these two cities?
(c) Hence, calculate the shortest distance between Beijing and Perth, to the nearest kilometre,
given that the Earth’s radius is 6370 km

56 by T.H Musondela
EXERCISE 17
1. Moscow, Russia and Copenhagen, Denmark have coordinates (55°N, 38°E) and (55°N,
12°E) respectively.
(a) What special small circle passes through both cities?
(b) What is the angle between Moscow and Copenhagen at the centre of this small circle?
(c) If the radius of this small circle is 3652 km, use the arc length formula to calculate the
small circle distance between Moscow and Copenhagen, to the nearest kilometre.

2. Melbourne (37°S, 145°E) and Cairns (17°S, 145°E) lie on the same meridian of longitude.
(a) Is the 145°E meridian a great circle or small circle?
(b) What is the size of the angle made by these two cities at the centre of this circle?
(c) Calculate the shortest distance between Melbourne and Cairns, to the nearest kilometre.

3. Perth (32°S, 116°E) and Taree (32°S, 152°E) lie on the same parallel of latitude.
(a) Is the 32°S parallel a great circle or small circle?
(b) What is the size of the angle made by these two cities at the centre of this circle?
(c) Calculate, to the nearest kilometre, the distance between Perth and Taree along this circle
if its radius is 5400 km.

4. Calculate, to the nearest kilometre, the great circle distances between these cities.
(a) Osaka, Japan (34°N, 135°E) and Alice Springs (23°S, 135°E)
(b) Pretoria, South Africa (25°S, 28°E) and Minsk, Belorussia (54°N, 28°E)
(c) Kampala, Uganda (0°, 32°E) and Macapa, Brazil (0°, 51°W)
(d) Mawson, Antarctica (67°S, 63°E) and Chelyabinsk, Russia (54°N, 63°E)
(e) Port Moresby, Papua New Guinea (9°S, 147°E) and Nyngan (31°S, 147°E)
(f) Geelong (38°S, 144°E) and Ivanhoe (33°S, 144°E)
(g) Leeton (34°S, 146°E) and Townsville (19°S, 146°E)

57 by T.H Musondela
EXERCISE 18
1.(a) Two cities, A and B, lie on the Equator, one at longitude 65°E and the other at
longitude 23°W. Calculate the distance between the two towns, measured along the Equator.
(b) . Two towns are located on longitude 50°E. The towns are 5 200 km apart. Calculate the
difference in their latitudes.
(c) . Find the length of the parallel of latitude 41°S.

2. The Antarctic Circle near the South Pole is the 66.5° S parallel of latitude.
(a) Calculate how many kilometres this circle is from: the Equator
(b) Calculate how many kilometres this circle is from the South Pole

3. Durban, South Africa has coordinates (30°S, 31°E) while Grafton has coordinates (30°S,
153°E). If the radius of the 30°S parallel of latitude is 5515 km, calculate the distance
between Durban and Grafton along this parallel, to the nearest kilometre.

4. If you dug a hole from Beijing, China (40°N, 116°N) through the centre of the Earth and
came out on the other side, you would end up at point X, somewhere in Argentina.
(a) What would be the coordinates of the point X?
(b) How many kilometres are there between Beijing and point X:
(i) in a straight line through the Earth’s centre? (ii) in a great circle along the Earth’s surface?

5. (a) The great circle distance between Sydney and Honolulu, Hawaii is 4170 km. What
angle is made at the centre of the Earth by these two cities? Answer to the nearest degree.
(b) It is possible to fly from Perth (32°S, 115°E) to Tierra del Fuego (54°S, 65°W) on the
southern tip of South America by going directly over the South Pole along a line of longitude.
Calculate the great circle distance between Perth and Tierra del Fuego, to the nearest
kilometre.

58 by T.H Musondela
EXERCISE 19
1. Two towns, C and D, lie on the same longitude. Their latitudes are 80°N and 70°S,
respectively. Calculate the shortest distance between the two towns.

2. Two places are located on the parallel of latitude 32°N, one at longitude 47°W and the
other at longitude 25°E.
a) How far apart are the two towns, measured along the parallel of latitude 32°N°
b) If it takes a plane 10 hours to travel from one town to the other, calculate the speed of the
plane, correct to the nearest kilometer per hour.

3.(a) Two towns, P and Q, are located at (4°N, 100°E) and (4°N, 140°E), respectively.
Calculate the shortest distance from P to Q, along the parallel of latitude 4°N.
(b) . Havana and Canton are both on latitude 23°N, and their longitudes are 82.25°W and
113.25°E, respectively. Find their distance apart measured along the parallel of latitude.

4. The points A (50°N, 30°E) and B (50°N, 150 °W) are two points on the surface of the
earth. The radius of the earth R = 6370 km or 3437 nautical miles.
(a) Find the distance between A and B along a parallel of latitude in:
(i) Kilometres (km) (ii) Nautical miles (nm)
(b) The shortest distance from A to B in nautical miles.
(c) An aircraft takes 54 hours to fly between the two towns A and B along the great circle.
Calculate its speed in knots correct to 2 significant figures.

5. A (50°S, 20°W) and B (50°S, 60°W) are two points on Earth’s surface. The radius of Earth
is approximately 6400 km.
(a) Neatly sketch the above information;
(b) Calculate the local time at A if it is 09 00 at B.
(c) Calculate the radius of the parallel of latitude 50 °S to the nearest kilometre.
(d) Calculate the shortest distance between A and B along the parallel of latitude 50°S.

59 by T.H Musondela
EXERCISE 20
1. The position of two towns X and Y are given to the nearest degree as X (45° N, 110° W)
and Y (45° N, 70° E). Take  = 3.142, R = 6370 km.
(a) Find the distance between the two towns along the parallel of latitude in km.
(b) Find the distance between the towns in nautical miles.
(c) A plane flew from X to Y taking the shortest distance possible. It took the plane 15hrs to
move from X and Y. Calculate its speed in Knots.
(d)Find the local time at Y when the local time at X on 10th April is 10.00pm.

2. Two planes P and Q are at (36°N, 125° W) and (36° N , 55° E) respectively.
Calculate the distance
(a) in nautical mile between P and Q measured along the circle through the North Pole.
(b) in nautical mile between P and Q measured along the common circle of latitude.

3. Two points on the Earth’s surface are P (13°N, 75°W) and Q (49°N, 75°W).
(a) What is the angular distance between P and Q°
(b) Calculate the distance, PQ, in nautical miles.
(c) Use 1 M = 1.852 km to give the distance, PQ, correct to the nearest 10 km.
(d) calculate the distance, PQ, correct to the nearest 10 km.

4. The coordinates of A are (20°S, 130°E) and the coordinates of B are (15°N, 130°E).

(a) Find the angular distance between them.

(a) Find the shortest distance between A and B measured on the surface of the earth.

5. (a) X and Y are two points on the Earth’s surface with coordinates (32°N, 120°E) and
(45°S, 120°E). Calculate the distance, XY, correct to the nearest 100 km

(b) Two points, A and B, on the Earth’s surface are at (30°N, 25°W) and (20°S, 25°W).
Calculate the angular distance between A and B.

60 by T.H Musondela
EXERCISE 21
1. X and Y are two points on the Earth’s surface with coordinates (32°N, 120°E) and (45°S,
120°E). Calculate the distance, XY, correct to the nearest 100 km.

2. Calculate the distance between each of the points below in nautical miles.
(a) A (10°N, 45°E) and B (25°S, 45°E) (b) C (75°N, 86°W) and D (60°S, 86°W)
(c) E (46°S, 52°W) and F (7°S, 52°W) (d) G (34°N, 172°E) and H (62°S, 172°E)

3. The city of Osaka is at (37°N, 135°E) while Alice Springs is at (23°S, 135°E).
(a) Calculate the distance between Osaka and Alice Springs in nautical miles.
(b) Use 1 M =1.852 km to write this distance, correct to the nearest kilometre.

4. The Tropic of Cancer is at latitude 22 °N while the Tropic of Capricorn is at latitude 22 °S.
Calculate the distance between these two tropics along the same great circle in (a) nautical
miles (b) kilometres (correct to the nearest km).

5. (a) M and N are two points on the Earth’s surface with coordinates (56°N, 122°W) and
(3°S, 122°W). Calculate the distance, MN, correct to the nearest 100 km, using the arc length
formula. (Take the radius of the Earth to be 6400 km.)
(b) . Find the distance between the point A 60°N, 50°E and the point B 60°N, 170°E to the
nearest kilometre.

8 .(a) (i) Draw a sketch of the earth and label on it the meridians of longitude.
(a) O˚ (b) 83˚E (c) 52˚W

(ii) What fraction of the earth’s surface area is covered by the region stretching Eastwards
from 52˚W to 83˚E.
(iii) Taking π=3.142, the radius of the earth (R) to be 6 370km and surface area of a sphere to
be 4πr2, calculate the surface area south of the equator stretching Eastwards from 52˚W to
83˚E, giving your answer in standard form correct to 3 significant figures.

61 by T.H Musondela
EXERCISE 22
1. A pane leaves an airport P (10°S, 60° E) and flies due north at 800km/hr. By taking radius
of the earth to be 6370 km and 1 nautical mile to be 1.853km,
(a) Find its position after 2hrs
(b) The plane turns and flies at the same speed due West to reach Q longitude 12°W. Find
the distance it has traveled due in West nautical miles
(c) Find the time it has taken
(d) If the local time at P was 1300 hrs when it reached Q. Find the local time at Q when it
landed at Q

2. Points R and S are two points on the surface on a latitude 48°S. The two points lie on
longitudes 30° W and 150°E respectively. By taking the earth’s radius to be 6370km,
calculate:
(a) The distance from R to S along a parallel of latitude.
(b) An aeroplane flies at an average speed of 2 80km/h from R to S along a great circle
through the South Pole. Calculate the total time taken.
(c) The local time of R when the local time of R is 14 15.
(d)Another point Q is 600Nm North of R .Find the location of Q

3. Two points A (70°N, 15oE) and B lie on the same circle of latitude on the earths surface.
Given that the shortest distance between the two points along the circle of latitude is 2133.6
22
km. Giving coordinates to the nearest degree, find the possible locations of B. (Take  = 7

and radius of earth = 6370km)

4 (a) Anchorage (60°N, 1 50°W) and Leningrad (60°N, 30°E) are two places on the surface of
the Earth.
(i) Find the difference in longitude between the two places,
(ii) Find the shortest distance between them, measured in nautical miles along the surface of
the earth.

62 by T.H Musondela
EXERCISE 23
1. A jet flies from town X (50oS, 20oE) directly to Y (50oS, 28oW) and then due south for
1200 nm to Z
(a) (i) Find the latitude of Z
(ii) Calculate the distance XY along a parallel of latitude 50oS in km
(b) (i) Given that the average speed of the jet is 400 knots, calculate the time taken to reach Z
from X to the nearest 0.1hour
ii) Find the time of arrival at Z given that the plane left X at 07 40. Take  = 22/7 and radius of
the earth to be 6370km

2. A jet on a rescue mission left town A (35oS, 15oE) to town B (45oN, 15oE) and then to
town C (45oN, 45oW). If 1o subtends 60nm and the radius of the earth is 6370km.
(a) Find the distance in nautical miles from A to C via B correct to 4 s.f
(b) Find the distance in kilometers from A to B to the nearest km
(c) Find the jet flew at 840km/h from A to C. If the jet left town A at 8.15a.m, what time
will it arrive at town C in local time

4. P, Q and R are points on the surface of the earth such that P (60°N, 20°W), Q (60°S,
20°W) and R (60°N, 80°E).
(a) Find the shortest distance between P and Q on the surface of the earth in kilometres and
nautical miles (nm)
(b) Find the length of latitude 60º N and hence the length of the minor arc PR in kilometres
(c) Find the distance from P to the North Pole

2. Town A is on latitude 24°S and longitude 118° W , while Town B is on latitude 24°S and
longitude 62°E. Take pie to 3.142 and R to 6370km
(a ) Sketch the Earth and indicate Town A and B.
( b) Calculate the distance between A and B along the circle of latitude.
( c) Calculate the distance between A and B via South pole.
(d ) An Aircraft took 2 hours to travel from A to B via South pole, calculate the speed.

63 by T.H Musondela
EXERCISE 24
1. A ship sailed due north for 630 nautical miles at a speed of 15 knots until it reached a port
on the equator. It then returned to its starting point, traveling due south at constant speed. The
return journey took 28 hours.
(i) Find the latitude from which the ship first sailed
(ii) Find the speed of the ship on the return journey,
(iii) Find the average speed of the ship for the whale voyage.

2. P and Q are two places on the surface of the Earth both in latitude 50N. The longitude of P
is 1O°W and the longitude of Q is 170° E. An aircraft flew at a speed of 600 knots from P to
Q along the shortest route over the North Pole.
(i) Calculate the distance, in nautical miles, flown by the aircraft,
(ii) Calculate the time taken for the aircraft to reach Q,
(iii) Calculate the distance, in nautical miles, between P and Q measured along their common
circle of latitude.
(iv) A second aircraft flew from P to Q along the circle of latitude 50° N and departed 30
minutes after the first aircraft. Find the speed at which the second aircraft had to travel in
order that both aircraft arrived at Q at the same time.

3. Two towns A and B lie on the same parallel of latitudes 60° N. The longitudes of A and B
are 42° W and 29° E respectively and C is another town due South of A and 10010 km away
from A
(a) Find the distance between A and B in nautical miles along the parallel of latitude.
(b) Find the local time at A if at B is 1.00pm.
22
(c) Find the distance between A and B in km. Take = 7
and R = 6370km

(d) Find the coordinate of C.


4. When it is 9 pm in Bombay, India (19°N, 73°E), what is the local time in Jakarta, Indonesia
(6°S, 106°E)?

64 by T.H Musondela
EXERCISE 25
1. A ship in distress sends out an SOS signal and gives its position as (60°N, 30°W). The
signal is picked up by two other ships: ship A, in position (60°N, 25°W) and ship B, in
position (55°N, 30°W). If ship A can steam at 15 knots and ship B at 25 knots, which one will
arrive on the scene first? (Assume that they both start immediately after they receive the
signal and that A steams due W, B due N).

2. (a) A(65°N, 5°E), B(65°N, 45°W) and C are three points on the surface of the earth and 0
is the centre of the earth. The point C, due south of A, is such that 𝐴𝑂𝐶= 82°.
(i) State the longitude of C.
(ii) Calculate the latitude of C.
(iii) Calculate, in nautical miles, the shortest distance
(a) between A and C, measured along the common meridian,
(b) between A and B, measured along the circle of latitude.

3. Q is a point on the surface of the Earth. It lies on the equator and in longitude 50° W.
(i) An aircraft A took off from Q and flew due west for 3 hours at a constant ground speed of
540 knots, reaching the point P. Calculate the longitude of P.
(ii) Another aircraft B took off from Q, flew due south until it reached the point R (32° S, 50°
W), and then flew 2000 nautical miles due east along the parallel of latitude 32° S to reach the
point T.
(a) Calculate the total distance, in nautical miles, flown by the aircraft B,
(b) Calculate the latitude and longitude of T.
(iii) The point on the Earth which is furthest from R is the point F.
(a)Write down the latitude and longitude of F.
(b)Calculate, in nautical miles, the shortest distance, measured on the surface of the Earth,
between R and F

65 by T.H Musondela
EXERCISE 26
1. An aircraft flies due east from an airfield at A (28° N 80° E) towards another airfield B on
the same parallel of latitude but with longitude 113° E.
(a) After travelling for one-third of the distance from A to B the aircraft makes a forced
landing at C. State the latitude and longitude of C
(b)A helicopter sets out from a base at D (25° N, 86° E) to search for the aircraft. The
helicopter flies due north to a place P in latitude 28 N and then flies along the parallel of
latitude to C.
(i)State the latitude and longitude of P.
(ii) Calculate, in nautical miles, the distance flown from D to F,
(iii) Calculate, in nautical miles, the distance flown from P to C.

2. A(45°N, 70° W), B ( 45°N, 40° W), C (20°N, 40° W), D (20°N,70° W )are four points on
the earth’s surface . Two aero planes start from A-to fly to C. One flies along the circle of
latitude to B then due south to C; the other flies due south to D and then along the circle of
latitude to C.
(i) Find the total distance traveled by each,
(ii) Find the time of flight of the first aero plane if its speed is 420 knots,
(iii) Find the speed of the second aero plane if its time of flight is 7 h 36 minutes.

3.(a) An aero plane leaves Oslo (60° N, 11° E) and flies due north to a point directly over the
North Pole. From there it flies due south to land at Anchorage (60° N, 150° W).
(i) How long was the flight in nautical miles?
(ii) If the flight lasted 6 h 24 min, what was the average speed of the aero plane in knots?
(iii) If instead, the aero plane had flown from Oslo to Anchorage due west along the 60°
parallel of latitude at the same average speed, how long would it have taken, to the nearest
minute?
(b) A boat is sailing at a speed of 18 knots. Calculate, nautical miles, the distance it will
travel in 2 14 hours,

66 by T.H Musondela
EXERCISE 27
7. The Tropic of Cancer is a name for the 23 12 ° N parallel of latitude.
(i) Find, in nautical miles, its circumference
(ii) Find, in nautical miles, its distance from the equator measured along the meridian.
8. The distance over the North Pole between Anchorage (61° N, 150° W) and Murmansk (x°
N, 30° E) is 3000 nautical miles. Find x.
9. Calculate, in nautical miles, the distance along the meridian between the points
C (20°S, 15°W ) and D (50°N, 15°W ).
10. Calculate, in nautical miles, the distance along the circle of latitude between the points
F (24° 30’S, 47°W) and G (24° 30’ S, 72° W).
11. The point P lies on the circle of latitude k° N. Given that the circumference of this circle
is of length 15600 nautical miles, calculate the value of k.
12. A ship steams 200 n.m. due north from a position in latitude 1° 45’ S. Find its new
latitude.
13. A second ship steams 200 n.m. due east from a position (60° N, 179° E). Find its new
Longitude.
14. A Boeing 747SP Jumbo Jet flies from Tokyo (140°E, 37°N) to Athens (123°W, 37°N)
carrying sportsmen for 2007 Olympics. It flies along the parallel of latitude at an average
speed of 450 knots. How long was the flight?
15 A ship leaves A (25° S, 61°E) and sails due east at a constant speed of 22 knots. After 14
hours the ship reaches a point B. Calculate, correct to the nearest minute, the longitude of B.
16. A ship sails due west from San Francisco (37° 47 ’ N, 122° 26’ W) for a distance of 1320
km.Calculate the longitude of its new position.
17. A boat is sailing on a bearing of 019° and the captain orders the course to be changed by
24°. Find the two possible bearing on which the boat might then sail.
18. In this question take the earth to be a sphere of radius 6371 km and  to be 3142. A
satellite orbits the earth on a circular path 229 km above the equator at a speed of 28000
km/h. Calculate the time taken for the satellite to circle the earth once, giving your answer
correct to the nearest minute.

67 by T.H Musondela
EXERCISE 28
1 . Philadelphia and Reno are two cities on the same circle of latitude north of the equator.
The longitude of Philadelphia is 75° W and the longitude of Reno is 120 W.
(i) Given that the distance between the cities is 3830 km, measured along the circle of
latitude, find this latitude.
(ii) Clearwater Lake is 1900 km due north of Philadelphia and Dawson Creek is 1900 km due
north of Reno. Find the distance between Clearwater Lake and Dawson Creek measured
along their common circle of latitude

2. Two towns A and B lie on the same latitude in the northern hemisphere. When its 8 00 am
at A, the time at B is 11 00. The longitude of A is 15° E.
(a) Find the longitude of B.
1
(b) A plane leaves A for B and takes 32 hours to arrive at B travelling along a parallel of

latitude at 850km/h.
(i) Find the radius of the circle of latitude on which towns A and B lie.
(ii) Find the latitude of the two towns (take radius of the earth to be 6371km)

3. A plane leaves an airport A (38.5, 37.05° W) and flies dues North to a point B on latitude
52 N.
(a) Find the distance covered by the plane
(b) The plane then flies due east to a point C, 2400km from B. Find the position of C
Take the value  of as 22/7 and radius of the earth as 6370 km

4. A plane flying at 200 knots left an airport A (30° S, 31°E) and flew due North to an airport
B (30° N, 31°E)
(a) Calculate the distance covered by the plane, in nautical miles
(b) After a 15 minutes stop over at B, the plane flew west to an airport C (30°N, 13°E) at the
same speed. Calculate the total time to complete the journey from airport C, though airport B.

68 by T.H Musondela
EXERCISE 29
1. An aero plane flies at an average speed of 500 knots due East from a point p (53°W, 4° E )
to another point Q. It takes 2 ¼ hours to reach point Q.
(i) Calculate the distance in nautical miles it traveled;
(ii) Calculate the longitude of point Q to 2 decimal places

2 (a) Point P (40°S, 45°E) and point Q (40°S, 60°W) are on the surface of the Earth.
Calculate the shortest distance along a circle of latitude between the two points.
(b) Two places A and B are on the same circle of latitude north of the equator. The longitude
of A is 118° W and the longitude of B is 133° E. The shorter distance between A and B
measured along the circle of latitude is 5422 nautical miles.
Find, to the nearest degree, the latitude on which A and B lie .

3. In this question, assume the earth to be a sphere of radius 6 370 km and use 𝜋 = 3.142.
The latitudes and longitudes of Antigua and Belize which are in different countries are

Country Latitude Longitude


Antigua 17°N 62°W
Belize 17°N 88°W

(i) Draw a sketch of the earth showing the location of Antigua (A) and of Belize (B), their
associated circles of latitude and longitude, the equator, and the Greenwich Meridian.
(ii) Calculate the shortest distance between Antigua and Belize measured along their common
circle of latitude.
(iii) A town, Bahia Bianka (BB) , situated in South America, lies on a meridian 62°W and has
latitude of 38°S.
(a) Calculate the shortest distance between Antigua and Bahia Bianka measured along the
common circle of longitude.
(b) Find the time at Belize when it is 09 35 at Bahia Bianka.

69 by T.H Musondela

Potrebbero piacerti anche